Study**Chapter 9: Drug Therapy for Coagulation Disorders

¡Supera tus tareas y exámenes ahora con Quizwiz!

A nurse is caring for a client who is receiving heparin by continuous IV infusion. The client begins vomiting blood. After the heparin has been stopped, which of the following medications should the nurse prepare to administer? A) Vitamin K B) Atropine C) Protamine D) Calcium gluconate

C) Protamine

The nurse instructs a client with a history of atrial fibrillation on warfarin. Which client statement indicates that teaching about the medication has been effective? "I will take a dose when I begin to feel fatigued." "I will avoid walking barefoot in the house and outdoors." "I will use a regular razor when shaving." "I will have the blood level of the drug checked twice a year."

"I will avoid walking barefoot in the house and outdoors." Explanation: Warfarin is an oral anticoagulant agent that is used to treat cardiovascular health problems. Because the medication thins the blood, the client should be instructed to avoid walking barefoot in the home and outdoors to reduce the risk for injury. The client should be instructed to use an electric razor when shaving to limit the risk for cuts. Warfarin is not a stimulant and is to be taken routinely as prescribed. Routine blood testing is necessary to ensure that the dose is appropriate. At first, this testing might be performed weekly. However, once the dose stabilizes, the blood tests are done less frequently, such as every two weeks.

The nurse educates a client diagnosed with atrial fibrillation about the prescribed warfarin. Which client statements establish the need for further clarification? "I will take the warfarin at the same time each day." "I will not drink alcohol unless it is all right with my primary care provider." "I will stop eating green leafy vegetables, broccoli, yogurt, or cheese." "I will keep my lab appointments for prothrombin and INR levels."

"I will stop eating green leafy vegetables, broccoli, yogurt, or cheese." Explanation: The nurse needs to clarify teaching regarding the intake of foods rich in vitamin K. The client should not avoid eating foods with vitamin K, but eat consistent amounts every day so therapeutic levels for the warfarin may be maintained. The client has an adequate understanding of the importance of keeping lab appointments for PT and INR levels used to monitor and regulate the warfarin dosage. The client should take the warfarin the same tie each day to maintain consistent blood levels of the drug. The client should not drink alcohol unless the primary care provider approves it, and instructs the client as to how much and how often. Alcohol will increase the risk for bleeding.

A client recovering from joint replacement surgery is prescribed subcutaneous heparin injections. Which response will the nurse make when the client asks if the medication can be taken orally? "The medication has to be given through an injection because your activity status is limited." "It will take too long for the medication to work if it is given by mouth." "The gastrointestinal tract does not absorb the drug, so it cannot be given by mouth." "The medication needs to be given by injection because of the type of surgery that you had."

"The gastrointestinal tract does not absorb the drug, so it cannot be given by mouth." Explanation: It is necessary to give heparin intravenously or through subcutaneous injection because the gastrointestinal tract does not absorb the drug. There is not an oral form of heparin. The route for administering heparin is not determined by the activity status or the type of surgery.

Which client statement demonstrates an accurate understanding of the action of a prescribed anticoagulant? "This medication will keep unnecessary clots from forming in my blood." "If a big clot starts to form, this medication will help break it up." "Anticoagulants thin the blood so clots can't get as big." "If I have another stroke, this medication will bust up the clot before too much damage is done."

"This medication will keep unnecessary clots from forming in my blood." Explanation: An anticoagulant slows the body's normal blood-clotting processes to prevent harmful blood clots from forming. This type of drug is often called a "blood thinner"; however, it cannot dissolve any clots that have already formed and does not make your blood thin.

A patient with a congenital coagulation disorder is given a hemostatic to stop bleeding following surgery. The nurse will carefully monitor this patient for development of which adverse effects? (select all that apply) 1. anaphylaxis 2. hypertension 3. hemorrhage 4. headache 5. hypotension

1. anaphylaxis 4. headache 5. hypotension

What patient education should be included for a patient receiving heparin? (Select all that apply) 1. teach the patient to give subcutaneous injections at home 2. teach the patient not to take any over-the-counter drugs without first consulting with the healthcare provider 3. teach the patient to observe for unexplained bleeding, such as pink, red, or dark brown urine or bloody gums 4. teach the patient to monitor for the development of deep vein thrombosis 5. teach the patient about the importance of drinking grapefruit juice daily

1. teach the patient to give subcutaneous injections at home 2. teach the patient not to take any over-the-counter drugs without first consulting with the healthcare provider 3. teach the patient to observe for unexplained bleeding, such as pink, red, or dark brown urine or bloody gums 4. teach the patient to monitor for the development of deep vein thrombosis

A client receives subcutaneous heparin at 8 am and is scheduled for the next dose at 8 pm. The nurse would expect to obtain a specimen for an aPTT test at which time? 1700 1400 1800 1200

1400 Explanation: When administering heparin by the subcutaneous route, an aPTT test is performed 4 to 6 hours after the injection. Less than 4 to 6 hours or more than 4 to 6 hours will not have the optimal peak value.

The patient is receiving heparin therapy and asks how the "blood thinner" works. What is the best response by the nurse? 1. "Heparin makes the blood less thick." 2. "Heparin does not thin the blood but prevents clots from forming as easily in the blood vessels." 3. "Heparin decreases the number of platelets so that blood clots more slowly." 4. "Heparin dissolves the clot."

2. "Heparin does not thin the blood but prevents clots from forming as easily in the blood vessels."

A patient has started clopidogrel after experiencing a transient ischemic attack. What is the desired therapeutic effect of this drug? 1. anti-inflammatory and antipyretic effects 2. to reduce the risk of a stroke from a blood clot 3. analgesic as well as clot-dissolving effects 4. to stop clots from becoming emboli

2. to reduce the risk of a stroke from a blood clot

A client is receiving anticoagulant therapy and has an INR done. Which result would the nurse interpret as being therapeutic? 1.4 2.4 1.8 1.0

2.4 Explanation: When anticoagulant therapy is used, the INR is maintained between 2 and 3. The other values are not therapeutic.

A patient with deep vein thrombosis is receiving an infusion of heparin and will be started on warfarin soon. While the patient is receiving herapin, what laboratory test will provide the nurse with information about its therapeutic effects? 1. Antithrombin III 2. International normalized ratio (INR) 3. Activated partial thromboplastin time (aPTT) 4. Platelet count

3. Activated partial thromboplastin time (aPTT)

A patient is receiving a thrombolytic drug, alteplase, following an acute myocardial infarction. Which effect is most likely attributed to this drug? 1. skin rash with urticaria 2. wheezing with labored respirations 3. bruising with epistaxis 4. temperature elevation of 38.2 F

3. bruising with epistaxis

When administering tirofiban, the nurse would administer the drug intravenously over which time frame? 15 minutes 60 minutes 30 minutes 45 minutes

30 minutes Explanation: Tirofiban is administered intravenously over 30 minutes.

A client receiving heparin has a baseline aPTT of 28 seconds. Which result would the nurse interpret as suggesting too much heparin? 42 seconds 70 seconds 38 seconds 98 seconds

98 seconds Explanation: Therapeutic range for heparin would be 1.5 to 2.5 times the client's baseline. For a baseline value of 28 seconds, this would range from 42 to 70 seconds. A value of 98 seconds, which is 3.5 times the client's baseline, would indicate too much heparin.

The client is learning to self-administer heparin daily. The home care nurse draws blood every 2 to 3 days to monitor the client's platelet levels. What platelet count would the nurse report to the health care provider as a critical laboratory result? 160,000 90,000 150,000 400,000

90,000 Explanation: Daily visits by a home care nurse may be needed if the client or a family member is unable or unwilling to inject the medication. Platelet counts should be obtained before therapy and every 2 to 3 days during heparin therapy. Heparin should be discontinued if the platelet count falls to less than 100,000 or to less than half the baseline value.

Heparin therapy is prescribed to a client with deep vein thrombosis. Which client would be at greatest risk of bleeding from this therapy? A 31-year-old with a history of DIC A 45-year-old corporate executive with peptic ulcer disease A 62-year-old with a history of prior cerebral vascular accident 5 years ago A 25-year-old with a history of hypertension

A 45-year-old corporate executive with peptic ulcer disease Explanation: Heparin is also contraindicated in clients with thrombocytopenia, bleeding disorders, and active bleeding other than DIC. This drug should be used with caution in clients with the potential for hemorrhage (e.g., immediately after surgery, with peptic ulcer disease, and with liver disease).

The nurse notes that a client receiving warfarin​ (Coumadin) has a high international normalized ratio​ (INR). Which question should the nurse include in the​assessment? A. ​"Do you drink a lot of​ milk?" B. ​"Do you eat a lot of​ garlic?" C. ​"Are you restricting your​ fluids?" D. ​"Have you been eating a lot of salads and​ vegetables?"

B. ​"Do you eat a lot of​ garlic?"

Which client would be the best candidate to receive alteplase recombinant therapy? A 72-year-old female who had a hemorrhagic stroke A 14-year-old male diagnosed with hepatic impairment An 18-year-old female who has intracranial bleeding A 68-year-old male who has had an ischemic stroke that resulted in neurological deficits

A 68-year-old male who has had an ischemic stroke that resulted in neurological deficits Explanation: Alteplase recombinant is the only drug approved for treating ischemic stroke. After a CT scan rules out intracranial bleeding or hemorrhagic stroke, treatment should be initiated within 3 hours after the onset of the stroke. The drug can be used only for patients who have a clinical diagnosis of stroke with clinically meaningful neurologic deficit because its adverse effects can be fatal. Alteplase recombinant may also be used to treat massive pulmonary embolism, but caution is necessary in patients with significant hepatic impairment.

The nurse is preparing to assess a client with von​ Willebrand's disease​ (vWD). Which priority question should the nurse ask the​ client? A. ​"What are you currently using for birth​ control?" C. ​"Do you have any other medical​ conditions?" D. ​"Have you ever been​ pregnant?" B. ​"Do you have heavy menstrual​ periods?"

B. ​"Do you have heavy menstrual​ periods?"

What intervention does the nurse include in the plan of care for a client receiving a continuous intravenous infusion of heparin? A) Avoid IM injections B) Assessing for symptoms of respiratory depression C) Measuring hourly urinary outputs D) Monitoring BP hourly

A) Avoid IM injections (Avoid all invasive procedures, including giving IM injections, while the client is on heparin therapy.)

What is the action of anticoagulants? A) Decrease blood coagulability- no effect on existing thrombus B) Prevent platelet aggregation at the site of blood vessel injury C) Destroy thrombi that are blocking blood vessels D) Prevent destruction of fibrin- promote clot formation

A) Decrease blood coagulability- no effect on existing thrombus

Which information should the nurse include the teaching for a client prescribed​ heparin? ​Note: Credit will be given only if all correct choices and no incorrect choices are selected. Select all that apply. A. ​Self-administration of subcutaneous injections. B. Symptoms of deep vein thrombosis. C. Required laboratory tests. D. Signs of abnormal bleeding. E. Scheduled administration times with meals.

A. ​Self-administration of subcutaneous injections. B. Symptoms of deep vein thrombosis. C. Required laboratory tests. D. Signs of abnormal bleeding.

The FDA has issued a black box warning for the use of protamine sulfate due to the risk of which conditions? (Select all that apply.) A) Severe hypotension B) Cardiovascular collapse C) Cardiogenic pulmonary edema D) Pulmonary hypertension E) Pulmonary vasodilation

A) Severe hypotension B) Cardiovascular collapse D) Pulmonary hypertension

Which describes the mechanism of action of anticoagulant​ prescriptions? A. Inhibition of thrombi formation. B. Conversion of plasminogen to plasmin C. Alteration of plasma membrane and platelets D. Prevention of fibrin from dissolving

A. Inhibition of thrombi formation.

Which anatomical area should the nurse display a client when providing education about the point of origin of a pulmonary​ embolism? A. Right atrium B. Left atrium C. Right ventricle D. Left ventricle

A. Right atrium

Which food should the nurse instruct the client prescribed warfarin​ (Coumadin) to​ avoid? A. Tomato salad with kale and basil B. Fettuccine Alfredo C. ​Whole-wheat bread with margarine D. Salt substitute

A. Tomato salad with kale and basil

The nurse has provided discharge education for a client prescribed an anticoagulant. Which statement made by the client indicates an understanding of the​information? A. ​"I should wear a medical alert bracelet that says​ I'm on an​ anticoagulant." B. "I can take​ enteric-coated aspirin but not plain aspirin for my​ arthritis." C. "I need to eat more protein while I am taking this​ medication." D. "I must limit my intake of vitamin C while​ I'm on warfarin​ (Coumadin)."

A. ​"I should wear a medical alert bracelet that says​ I'm on an​ anticoagulant."

Which statements should the nurse include in the teaching for a client prescribed warfarin​ (Coumadin) therapy? ​Note: Credit will be given only if all correct choices and no incorrect choices are selected. Select all that apply. A. ​"Inform your dentist you are taking warfarin prior to any​ procedures." B. ​"Report to the lab for testing of activated partial thromboplastin time​ (APTT)." C. ​"Avoid strenuous​ activities." D. ​"Place ice at the injection site if stinging or burning​ occurs." E. ​"Take nonsteroidal​ anti-inflammatories (NSAIDs) for minor pain​ relief."

A. ​"Inform your dentist you are taking warfarin prior to any​ procedures." C. ​"Avoid strenuous​ activities."

A patient who received a copy of her genomic testing and is classed as a "poor metabolizer" of clopidogrel asks the nurse what it means. The nurse, understanding the variant response to CYP2C19 would be most correct in stating which of the following? A. "You metabolize the medication different from some individuals and may not receive the same benefit from drug therapy." B. You lack the CPY2C19 isozyme, so you will not need to take clopidogrel." C. You will be prescribed less of the medication than is normally necessary." D. "You are at a higher risk of bleeding from the drug, so you must be extra careful."

A. "You metabolize the medication different from some individuals and may not receive the same benefit from drug therapy." With poor metabolizers of clopidogrel, the benefits of the drug may not be adequate.

A client is prescribed enoxaparin (Lovenox) for treatment of a newly diagnosed left lower extremity deep vein thrombosis (DVT). What information should the nurse obtain related to the administration of this drug? A. Client's weight B. Serum aPTT C. Serum PT/INR D. Angiogram results

A. Client's weight

A nurse is caring for a client receiving warfarin (Coumadin) therapy. The nurse should monitor the client for use of which drugs? (Select all that apply.) A. Diuretics B. Steroids C. Nicotine D. Barbiturates E. Alcohol

A. Diuretics B. Steroids E. Alcohol

The patient develops a deep vein thrombosis. The nurse anticipates administration of which medication? A. Intravenous heparin B. Clopidogrel C. Vitamin K D. Protamine sulfate

A. Intravenous heparin (Intravenous heparin is indicated for rapid anticoagulant effect when a thrombosis occurs because of a deep vein thrombosis (DVT), pulmonary embolism (PE), or an evolving stroke. The effects of subcutaneous heparin take longer to occur. Clopidogrel (Plavix) is an antiplatelet drug that is mainly for prophylactic use in prevention of myocardial infarction or stroke. Vitamin K is the antidote for warfarin, and protamine is the antidote for heparin.)

During a cardiac catheterization and concurrent GP IIb/IIIa receptor antagonist administration, a client begins to bleed from the arterial access site of the catheterization. In what, if any, specific circumstance can the infusion continue? A. It can continue if bleeding can be controlled by pressure application. B. It can continue if the procedure is within 15 minutes of completion. C. It can't D. It must be stopped. E. It can continue if the client can tolerate Trendelenburg's position.

A. It can continue if bleeding can be controlled by pressure application. If bleeding occurs and cannot be controlled with pressure, the procedure must be terminated. No other options are accurately good practice under the described situation.

A client with a history of unstable angina is being prepared for percutaneous transluminal coronary angioplasty (PTCA). The nurse assessing the client should focus on identifying a history of which conditions/situations that are considered contraindications to this procedure? Select all that apply. A. Major trauma within the last 6 weeks B. Thrombocytopenia C. Uncontrolled hypertension D. Heparin use E. Diabetes mellitus

A. Major trauma within the last 6 weeks B. Thrombocytopenia C. Uncontrolled hypertension A PTCA involves the administration of abciximab. Contraindications for abciximab administration include active bleeding, thrombocytopenia, history of a serious stroke, surgery or major trauma within the previous 6 weeks, uncontrolled hypertension, or hypersensitivity to drug components. Abciximab is contraindicated in clients who have recently received an oral anticoagulant but not heparin. Diabetes is not considered a contraindication for the use of abciximab.

A nurse is preparing discharge teaching for a client who will be receiving enoxaparin (Lovenox) injection. What should the nurse include in the teaching plan? A. Teaching the client to self-administer subcutaneous injections B. Monitoring the client's lab tests for hepatic impairment C. Teaching the client to observe for symptoms of anemia D. Monitoring the client for development of deep vein thrombosis

A. Teaching the client to self-administer subcutaneous injections

A nurse is caring for a client who is receiving alteplase (Activase) for treatment of an acute episode of myocardial infarction (MI). For which findings in the client's history should the nurse be alert to determine the client's risk for bleeding? (Select all that apply.) A. Use of ibuprofen for mild arthritis B. Recent antibiotic therapy with erythromycin C. Intake of herbals such as ginger or garlic D. Recent trauma or surgery E. Use of SSRI such as fluoxetine

A. Use of ibuprofen for mild arthritis C. Intake of herbals such as ginger or garlic D. Recent trauma or surgery

In explaining the use of warfarin to a female patient, the nurse is correct in telling her which of the following regarding warfarin? (Select all that apply) A. Warfarin is a vitamin K antagonist B. Warfarin does not cross the placenta C. Warfarin is used for long-term anticoagulation therapy D. Warfarin is metabolized by the liver

A. Warfarin is a vitamin K antagonist C. Warfarin is used for long-term anticoagulation therapy D. Warfarin is metabolized by the liver Warfarin, a pregnancy category X medication is contraindicated during pregnancy because it crosses the placenta and may produce fatal fetal hemorrhage. The US FDA has issued a Black Box Warning for warfarin due to its risk of causing major or fatal bleeding.

Clopidogrel is indicated for which of the following? A. reduction of myocardial infarction, stroke, vascular death in patients with atherosclerosis B. adjunctive therapy to warfarin for deep thrombosis C. heparin-induced thrombocytopenia D. patients in whom bleeding is a consideration

A. reduction of myocardial infarction, stroke, vascular death in patients with atherosclerosis The thienopyridine adenosine diphosphate receptor antagonists, including clopidogrel, prasugrel, and ticlopidine are used for their antiplatelet activity. The drugs are indicated for reduction of myocardial infarction, stroke, and vascular death in clients with atherosclerosis and after coronary stent placement.

Which of the following assessment findings are positive outcomes in a patient receiving warfarin for the management of pulmonary embolism? (Select all that apply.) A. report of absence of chest pain B. respiratory rate of 18/min wit no use of accessory muscles C. absence of uncontrolled bleeding D. international normalized ratio serum level of 3.2 E. prothrombin time serum level of 18 seconds F. presence of hematuria

A. report of absence of chest pain B. respiratory rate of 18/min wit no use of accessory muscles C. absence of uncontrolled bleeding E. prothrombin time serum level of 18 seconds Positive outcomes with the use of warfarin for the management of pulmonary embolism include less or absent chest pain, or signs of respiratory difficult, absence of uncontrolled bleeding, and PT value of approximately 1.5 times the control. or 18 second, and INR rang on warfarin of 2.0-3.9. Hematuria or blood in the stools is an indication of adverse effect.

A patient arrived in the emergency department 2 hours after an acute ischemic stroke. The patient is given an intravenous (IV) injection of alteplase tPA. It is most important for the nurse to monitor what? (Select all that apply.) A.Bleeding B.Vital signs C.PT levels D.Allergic reactions E.Electrocardiogram

A.Bleeding B.Vital signs D.Allergic reactions E.Electrocardiogram (The nurse should monitor the patient receiving thrombolytics for adverse effects, such as bleeding, allergic reactions, and cardiac dysrhythmias. An increased heart rate with a decreased BP usually indicates blood loss from bleeding.)

A client with cirrhosis of the liver asks the nurse why they are at risk for bleeding. Which response should the nurse provide the​ client? A. ​"The liver is injured and unable to manufacture​ platelets." B. ​"The liver is breaking down your clotting factors too​ quickly." C. ​"The liver is injured and cannot make clotting​ factors." D. ​"The liver thickens your blood so it is less likely to​ clot."

C. ​"The liver is injured and cannot make clotting​ factors."

After teaching a group of students about hemostatic agents, the instructor determines that the teaching was successful when the students identify what as a systemic hemostatic agent? Recombinant thrombin Absorbable gelatin Aminocaproic acid Human fibrin sealant

Aminocaproic acid Explanation: Aminocaproic acid is the only systemic hemostatic agent available. Absorbable gelatin is a topical hemostatic agent. Human fibrin sealant is a topical hemostatic agent. Recombinant thrombin is a topical hemostatic agent.

A patient is recovering from surgery to replace her right hip. In the early postoperative phase, the nurse anticipates administration of which drug to prevent deep vein thrombosis? A. Dipyridamole B. Low-molecular-weight heparin C. Abciximab D. Anagrelide

B. Low-molecular-weight heparin (Low-molecular-weight heparins are derivatives of standard heparin and were introduced to prevent venous thromboembolism. The other drugs are platelet inhibitors. Antiplatelets are used to prevent thrombosis in the arteries by suppressing platelet aggregation.)

The client receiving heparin therapy asks how the medication works. Which response by the nurse is most accurate? A. "Heparin makes the blood less viscous, preventing further clotting." B. "Heparin dissolves the clot specifically in the affected blood vessels." C. "Heparin inhibits clotting factors, thereby preventing formation of new clots." D. "Heparin decreases the number of platelets, so that blood clots more slowly."

C. "Heparin inhibits clotting factors, thereby preventing formation of new clots."

4. A patient has been administered heparin to prevent thromboembolism development status postmyocardial infarction. The patient develops heparin-induced thrombocytopenia. Which of the following medications will be administered? A) Argatroban (Acova) B) Vitamin K C) Calcium gluconate D) Aminocaproic acid (Amicar)

Ans: A Feedback: Heparin-induced thrombocytopenia may occur in 1% to 3% of those receiving heparin and is a very serious side effect of heparin. In this patient, all heparin administration must be discontinued and anticoagulation managed with a direct thrombin inhibitor, such as argatroban. The patient is not administered vitamin K, calcium gluconate, or aminocaproic acid

5. A patient is receiving low molecular weight heparin to prevent thromboembolic complications. The nursing student asks the nursing instructor the reason why this treatment is given instead of heparin. What is the instructor's best explanation of the rationale for LMWH over heparin? A) "LMWH is associated with less thrombocytopenia than standard heparin." B) "LMWH is associated with stronger anticoagulant effects than standard heparin." C) "LMWH is given to patients who have a history of blood dyscrasia." D) "LMWH is more effective than standard heparin for patients with hypertension."

Ans: A Feedback: Low molecular weight heparins are associated with less thrombocytopenia than standard heparin. Low molecular weight heparin is not stronger than standard heparin. Low molecular weight heparin is administered cautiously in patients with blood dyscrasia and hypertension

8. A patient is admitted with thrombophlebitis and sent home on enoxaparin (Lovenox). Which statement indicates a good understanding of why enoxaparin is being administered? A) Enoxaparin inhibits the formation of additional clots. B) Enoxaparin eliminates certain clotting factors. C) Enoxaparin decreases the viscosity of blood. D) Enoxaparin will dissolve the existing clots

Ans: A Feedback: Low molecular weight heparins prevent the development of additional clots. They do not eliminate clotting factors. LMWHs do not dissolve the clot or decrease the viscosity of blood

2. A patient who has been treated with warfarin (Coumadin) after cardiac surgery is found to have an INR of 9.0. Which medication will be administered to assist in the development of clotting factors? A) Vitamin K B) Vitamin E C) Protamine sulfate D) Acetylsalicylic acid (Aspirin)

Ans: A Feedback: Vitamin K is the antidote for warfarin overdosage. In this case, the patient may be at the therapeutic level to control thrombus formation, but, due to the injury, it is important to control bleeding. Vitamin E is not used as an antidote for warfarin overdosage. Protamine sulfate is used as an antidote to heparin or low molecular-weight heparin. Acetylsalicylic acid (Aspirin) is used to decrease coagulation as a preventive measure for myocardial infarction

16. A patient who is recovering in hospital from a bilateral mastectomy has developed minor bleeding at one of her incision sites. During the process of clot formation, plasminogen will become part of a clot by which of the following means? A) By binding with fibrin B) By binding with platelets C) By activating plasmin D) By activating factor

Ans: A Feedback: When a blood clot is being formed, plasminogen, an inactive protein present in many body tissues and fluids, is bound to fibrin and becomes a component of the clot. Plasminogen does not bind to platelets, activate plasmin, or active factor VII

12. A patient asks the nurse what dose of acetylsalicylic acid (Aspirin) is needed each day for antiplatelet effects to prevent heart attacks. What dose is most appropriate to reduce platelet aggregation? A) 10 mg B) 30 mg C) 625 mg D) 1000 mg

Ans: B Feedback: A single dose of 300 to 600 mg or multiple doses of 30 mg inhibit cyclooxygenase in circulating platelets almost completely. The dose of 10 mg is too small. The doses of 625 mg and 1000 mg are too large

A nurse is preparing to administer an antiplatelet drug. Which medication would the nurse likely administer? Phytonadione Enoxaparin Clopidogrel Alteplase

Clopidogrel Explanation: Clopidogrel is an antiplatelet agent. Alteplase is a thrombolytic agent. Phytonadione is an anticoagulant antagonist. Enoxaparin is an LMWH anticoagulant.

17. A 55-year-old man has been diagnosed with coronary artery disease and begun antiplatelet therapy. The man has asked the nurse why he is not taking a "blood thinner like warfarin." What is the most likely rationale for the clinician's use of an antiplatelet agent rather than an anticoagulant? A) Antiplatelet agents do not require the man to undergo frequent blood work; anticoagulants require constant blood work to ensure safety. B) Antiplatelet agents are more effective against arterial thrombosis; anticoagulants are more effective against venous thrombosis. C) Antiplatelet agents are most effective in large vessels; anticoagulants are most effective in the small vessels of the peripheral circulation. D) Antiplatelet agents have fewer adverse effects than anticoagulants

Ans: B Feedback: Anticoagulants are more effective in preventing venous thrombosis than arterial thrombosis. Antiplatelet drugs are used to prevent arterial thrombosis. CAD has an arterial rather than venous etiology. The rationale for the use of antiplatelet agents in CAD is not likely related to the need for blood work or the presence of adverse effects.

20. A 50-year-old man has undergone a bunionectomy and has been admitted to the postsurgical unit. What aspect of the man's medical history would contraindicate the use of heparin for DVT prophylaxis? A) The man is morbidly obese. B) The man has a diagnosis of ulcerative colitis. C) The man had a myocardial infarction 18 months ago. D) The man has a diagnosis of type 2 diabetes mellitus

Ans: B Feedback: GI ulcerations contraindicate the use of heparin. Obesity, diabetes, and previous MI do not rule out the safe use of heparin

10. A patient is discharged from the hospital with a prescription of warfarin (Coumadin). Which of the following statements indicates successful patient teaching? A) "If I miss a dose, I will take two doses." B) "I will avoid herbal remedies." C) "I will eat spinach or broccoli daily." D) "I will discontinue my other medications."

Ans: B Feedback: Most commonly used herbs and supplements have a profound effect on drugs for anticoagulation. The patient should never double up on dosing related to a missed dose. The patient should avoid green leafy vegetables due to vitamin K. The patient should not discontinue his or her medications

3. A patient has a history of clot formation. She is scheduled for bowel resection due to colorectal cancer. What anticoagulant agent will be administered prophylactically? A) Acetylsalicylic acid (Aspirin) B) Heparin C) Warfarin (Coumadin) D) Streptokinase (Streptase)

Ans: B Feedback: Prophylactically, low doses of heparin are given to prevent thrombus formation in patients having major abdominal surgery. Acetylsalicylic acid is not used to prevent thrombus in patients having major abdominal surgery. Warfarin takes several days for therapeutic effects to occur; thus it is not used prophylactically to prevent thrombus in a patient with abdominal surgery. Streptokinase promotes thrombolysis and is not used to prevent thrombus

7. A patient has experienced the formation of clots and has bruising. It is determined that there is a depletion of the patient's coagulation factors and widespread bleeding. Which of the following medications will be administered? A) Aminocaproic acid (Amicar) B) Heparin C) Warfarin (Coumadin) D) Protamine sulfate

Ans: B Feedback: The development of clots and widespread bleeding is indicative of disseminated intravascular coagulation. The patient should be administered heparin to slow the formation of clots. The goal of heparin therapy in DIC is to prevent blood coagulation long enough for clotting factors to replenish and control hemorrhage. Aminocaproic acid is used to control excessive bleeding from systemic hyperfibrinolysis. Warfarin is administered orally to decrease clot formation. Protamine sulfate would not be administered

11. A patient is being administered heparin IV and has been started on warfarin (Coumadin). The patient asks the nurse why she is taking both medications. What is the nurse's most accurate response? A) "After a certain period of time, you must start warfarin and heparin together." B) "You will need both warfarin and heparin for several days." C) "Warfarin takes 3 to 5 days to develop anticoagulant effects, and you still need heparin." D) "Warfarin cannot be given without heparin due to the amount of clotting you need.

Ans: C Feedback: Anticoagulant effects do not occur for 3 to 5 days after warfarin is started because clotting factors already in the blood follow their normal pathway of elimination. The statement "After a certain period of time, you must start warfarin and heparin together" does not explain clearly the reason for the two medications concurrently. The statement "You will need both warfarin and heparin for several days" does not explain clearly the reason for the two medications. The statement "Warfarin cannot be given without heparin due to the amount of clotting you need" is not accurate

15. A patient is taking warfarin (Coumadin) to prevent clot formation related to atrial fibrillation. How are the effects of the warfarin (Coumadin) monitored? A) RBC B) aPTT C) PT and INR D) Platelet count

Ans: C Feedback: The warfarin dose is regulated according to the INR. The INR is based on the prothrombin time. The red blood cell count is not indicative of warfarin dosage. The aPTT is utilized to determine heparin dose. The platelet count is required to determine warfarin dose

19. A 79-year-old woman has been brought to the emergency department by ambulance with signs and symptoms of ischemic stroke. The care team would consider the STAT administration of what drug? A) Low molecular weight heparin B) Vitamin K C) Clopidogrel (Plavix) D) Alteplase (Activase)

Ans: D Feedback: Alteplase (Activase) is used as first-line therapy for the treatment of acute ischemic stroke in selected people. Vitamin K would exacerbate the woman's symptoms, and LMWH and Plavix would be ineffective

13. A patient is prescribed eptifibatide (Integrilin), which inhibits platelet aggregation by preventing activation of GP IIb/IIIa receptors on the platelet surface and the subsequent binding of fibrinogen and von Willebrand factor to platelets. Which of the following syndromes are treated with eptifibatide? A) Blocked carotid arteries B) Intermittent claudication C) Hypertension D) Unstable angina

Ans: D Feedback: Eptifibatide (Integrilin) inhibits platelet aggregation by preventing activation of GP IIb/IIIa receptors on the platelet surface and the subsequent binding of fibrinogen and von Willebrand factor to platelets. Eptifibatide is used for acute coronary syndromes, including unstable angina, myocardial infarction, and non-Q wave MI. Blocked carotid arteries, intermittent claudication, and hypertension are not treated with eptifibatide

18. A nurse at a long-term care facility is conducting a medication reconciliation for a man who has just moved into the facility. The man is currently taking clopidogrel (Plavix). The nurse is most justified is suspecting that this man has a history of A) hemorrhagic cerebrovascular accident. B) hemophilia A. C) idiopathic thrombocytopenic purpura (ITP). D) myocardial infarction

Ans: D Feedback: Indications for use of Plavix include reduction of myocardial infarction, stroke, and vascular death in patients with atherosclerosis and in those after placement of coronary stents. It is not indicated in the treatment of ITP, CVA, or hemophilia

9. A patient is being discharged from the hospital with warfarin (Coumadin) to be taken at home. Which of the following foods should the patient be instructed to avoid in his diet? A) Eggs B) Dairy products C) Apples D) Spinach

Ans: D Feedback: Spinach is a green leafy vegetable that is high in vitamin K and will interact to prevent adequate levels of anticoagulant therapy. Eggs, dairy products, and apples are not contraindicated with warfarin

1. A patient is taking warfarin (Coumadin) after open heart surgery. The patient tells the home care nurse she has pain in both knees that began this week. The nurse notes bruises on both knees. Based on the effects of her medications and the complaint of pain, what does the nurse suspect is the cause of the pain? A) Joint thrombosis B) Torn medial meniscus C) Degenerative joint disease caused by her medication D) Bleeding

Ans: D Feedback: The main adverse effect of warfarin (Coumadin) is bleeding. The sudden onset of pain in the knees alerts the nurse to assess the patient for bleeding. Arthritis, torn medical meniscus, and degenerative joint disease could all be symptoms of knee pain, but the onset and combination of anticoagulant therapy is not an etiology of these types of injuries and disease

14. A patient is receiving IV heparin every 6 hours. An activated partial thromboplastin time (aPTT) is drawn 1 hour before the 08:00 dose. The PTT is 92 seconds. What is the most appropriate action by the nurse? A) Give the next two doses at the same time. B) Give the dose and chart the patient response. C) Check the patient's vital signs and give the dose. D) Hold the dose and call the aPTT result to the physician's attention

Ans: D Feedback: The normal control value is 25 to 35 seconds; therefore, therapeutic values are 45 to 70 seconds, approximately. A result of 92 seconds is a risk for bleeding, and the dose should be held until approval to administer is provided by the physician. The nurse should not give the next two doses at the same time. The nurse should not give the dose and document the patient's response. The nurse should not check the patient's vital signs and give the dose.

6. A patient who is receiving warfarin (Coumadin) has blood in his urinary catheter drainage bag. What medication will likely be ordered by the physician? A) Aminocaproic acid (Amicar) B) Platelets C) Protamine sulfate D) Vitamin K

Ans: D Feedback: Vitamin K is the antidote for warfarin overdose. Aminocaproic acid is used to control excessive bleeding from systemic hyperfibrinolysis. Platelets are a blood product, not a medication. Protamine sulfate is the antidote for heparin therapy

The most common adverse effects associated with antiplatelet agents are headache and dizziness. a) True b) False

Answer: True Rationale: Increased bruising and bleeding from the gums while brushing the teeth are the most common adverse effects associated with antiplatelet agents.

A nurse will use extreme caution when administering heparin to which of the following patients? a) A 38-year-old male with peptic ulcer disease b) A 22-year-old female with urticaria c) A 56-year-old male who smokes and drinks alcohol occasionally d) A 54-year-old female with accelerated heart rate

Answer: a) A 38-year-old male with peptic ulcer disease Rationale: Heparin should be administered with extreme caution to patients with peptic ulcer or liver disease or after surgery because those patients would have greater risk for hemorrhage or excessive blood loss. Urticaria is a listed adverse effect of taking heparin and would therefore be expected. A patient who takes heparin would be advised to not drink heavily or smoke, but neither would have the safety risk posed by a patient with peptic ulcer. A patient with an accelerated heart rate and on heparin therapy would be at no special risk.

Heparin is the anticoagulant of choice during lactation. a) False b) True

Answer: b) True Rationale: Although some adverse fetal affects have been reported with its use during pregnancy, heparin does not enter breast milk, and so it is the anticoagulant of choice if one is needed during lactation.

A nurse is preparing to administer the first dose of warfarin to a client. Which assessment should the nurse prioritize for this client? Assess PT and INR. Monitor for hypersensitivity reaction. Assess for signs of bleeding. Observe for signs of thrombus formation.

Assess PT and INR. Explanation: The nurse should assess the prothrombin time (PT) and INR before administering the anticoagulant drug warfarin to the client. Observing for signs of thrombus formation, assessing for signs of bleeding, and monitoring for hypersensitivity reaction are the ongoing assessments performed in clients who are administered warfarin.

A nurse is caring for a patient receiving the anticoagulant drug warfarin. What pre-administration assessments should the nurse perform before administering the drug to the patient? Observe for signs of thrombus formation. Assess prothrombin time (PT) and INR. Assess for signs of bleeding. Monitor for hypersensitivity reaction.

Assess prothrombin time (PT) and INR. Explanation: The nurse should assess the prothrombin time (PT) and INR before administering the anticoagulant drug warfarin to the patient. Observing for signs of thrombus formation, assessing for signs of bleeding, and monitoring for hypersensitivity reaction are the ongoing assessments performed in patients who are administered warfarin.

A nurse is monitoring a client who takes aspirin daily. The nurse should identify which of the following manifestations as adverse effects of aspirin? SELECT ALL THAT APPLY. A) Hypertension B) Coffee-ground emesis C) Tinnitus D) Paresthesias of the extremeties E) Nausea

B) Coffee-ground emesis C) Tinnitus E) Nausea

A nurse is planning to administer subq enoxaparin 40 mg using a prefilled syringe of 40 mg/0.4 mL to an adult client following hip arthroplasty. Which of the following actions should the nurse plan to take? A) Expel the air bubble from the prefilled syringe before injecting. B) Insert the needle completely into the client's tissue. C) Administer the injection in the client's thigh. D) Aspirate carefully after inserting the needle into the client's skin.

B) Insert the needle completely into the client's tissue.

A nurse is preparing to administer IV alteplase to a client. Which of the following interventions should the nurse plan to take? A) Administer IM enoxaparin along with the alteplase dose. B) Obtain the client's weight. C) Administer aminocaproic acid IV prior to alteplase infusion. D) Prepare to administer alteplase within 8 hours of manifestation onset.

B) Obtain the client's weight.

What is the action of antiplatelets? A) Decrease blood coagulability- no effect on existing thrombus B) Prevent platelet aggregation at the site of blood vessel injury C) Destroy thrombi that are blocking blood vessels D) Prevent destruction of fibrin- promote clot formation

B) Prevent platelet aggregation at the site of blood vessel injury

Which should the client be instructed to avoid when prescribed an​ anticoagulant? ​Note: Credit will be given only if all correct choices and no incorrect choices are selected. Select all that apply. A. Citrus fruits B. Alcohol C. Contact sports D. Prolonged sitting E. Hard toothbrush

B. Alcohol C. Contact sports D. Prolonged sitting E. Hard toothbrush

Which describes the mechanism of action for Clopidogrel​ (Plavix)? A. Stimulates platelet production B. Prevents the platelets from sticking together C. Prevents platelets from adhering to the injured tissue D. Decreases platelet production

B. Prevents the platelets from sticking together

The nurse is caring for a client with a DVT​ (deep vein​ thrombosis) receiving heparin intravenously​ (IV). Which is the priority outcome for the​ client? A. The client will comply with dietary restrictions. B. The client will not experience bleeding. C. The client will keep the right leg elevated on two pillows. D. The client will not disturb the intravenous infusion.

B. The client will not experience bleeding.

The nurse is preparing to provide education for a client prescribed Clopidogrel​ (Plavix) after a myocardial infarction and stent placement. Which statements should the nurse use to explain the action of​ Clopidogrel? ​Note: Credit will be given only if all correct choices and no incorrect choices are selected. Select all that apply. A. ​"Clopidogrel will dissolve any clots that might form in your​ stent." B. ​"Clopidogrel will make the platelets in your blood less​ sticky." C. ​"Clopidogrel will change the way your platelets​ work." D. ​"Clopidogrel decreases your​ blood's ability to​ clot." E. ​"Clopidogrel works just like the heparin you were prescribed when in the​ hospital."

B. ​"Clopidogrel will make the platelets in your blood less​ sticky." C. ​"Clopidogrel will change the way your platelets​ work." D. ​"Clopidogrel decreases your​ blood's ability to​ clot."

An older adult client with a history of mild renal failure has experienced two different transient ischemic attacks (TIAs) and has been prescribed clopidogrel therapy. What statement should be the basis of the nurse's response when asked why aspirin (ASA) A. Aspirin is very irritating to the stomach. B. Clopidogrel is believed to be more effective than aspirin in treating TIAs. C. Aspirin is not effective in treating TIAs. D. Clopidogrel is better suited for older clients with renal impairment.

B. Clopidogrel is believed to be more effective than aspirin in treating TIAs. Aspirin has long been the most widely used antiplatelet drug for prevention of myocardial reinfarction and arterial thrombosis in clients with TIAs and prosthetic heart valves. However, clopidogrel may be more effective than aspirin. While ASA can cause stomach irritation, that is not the reason it is not the drug of choice for this client. Because clopidogrel is metabolized in the liver, it may accumulate in people with hepatic impairment, but there is currently no contraindication in clients with renal failure.

The nurse, providing postprocedural care for a client scheduled for a cardiac catheterization, will plan to implement which intervention to manage the risk for a common adverse effect of the procedure? A. Initiation of cardiac pulmonary resuscitation B. Direct pressure to arterial access site C. Administration of intravenous heparin D. Intravenous administration of either eptifibatide or tirofiban

B. Direct pressure to arterial access site Bleeding is the most common adverse effect, with most major bleeding occurring at the arterial access site for cardiac catheterization. If bleeding occurs and cannot be controlled with pressure, the drug infusion and heparin should be discontinued. It is likely that either eptifibatide or tirofiban will be used during the procedure and would have no purpose in controlling bleeding in the postrecovery phase of a cardiac catheterization.

The nurse is caring for a client in long-term care who is receiving warfarin. Which assessment indicates a need to withhold the drug and consult with the client's primary health care provider? A. INR exceeds 1.0 B. INR exceeds 3 C. PT is 1.5 D. PT is 18 seconds

B. INR exceeds 3 Periodic assessment is necessary for clients on warfarin therapy to maintain safe and adequate drug levels. If the prothrombin time (PT) exceeds 1.2 to 1.5 times the control value or 18 seconds, or the international normalized ratio (INR) exceeds 3, the primary health care provider is notified before the drug is given.

The patient is being discharged home on warfarin therapy. Which information will the nurse include when teaching the patient? A. Results of activated partial thromboplastin time (aPTT) will determine if the medication is effective. B. International normalized ratio (INR) results should be between 2 and 3. C. A normal response to warfarin is for your stools to look tarry. D. Increase the amount of green leafy vegetables in your diet.

B. International normalized ratio (INR) results should be between 2 and 3. (Today, the international normalized ratio (INR) is the laboratory test most frequently used to report PT results; a value of 2 to 3 is considered acceptable. Partial thromboplastin time (PTT) and activated partial thromboplastin time (aPTT) are laboratory tests to detect deficiencies of certain clotting factors, and these tests are used to monitor heparin therapy. Tarry stools indicate GI bleeding. Green leafy vegetables contain vitamin K, which is the antidote for warfarin (Coumadin).)

An older adult client has been admitted to the emergency department with severe chest pain. Onset of symptoms is within the last 60 minutes. What medication would the nurse expect the health care provider to prescribe for this acute disorder? A. Direct thrombin inhibitor drugs B. Thrombolytic drugs C. Antiplatelet drugs D. Anticoagulant drugs

B. Thrombolytic drugs The scenario suggests that the client's chest pain may be a result of a thromboembolism. The main use of thrombolytic agents is for management of acute, severe thromboembolic disease, such as myocardial infarction or pulmonary embolism, since they dissolve blood clots. None of the other options can dissolve an existing blood clot.

In developing a safe plan of care. the nurse should recognize that the antidote for warfarin is A. protamine zinc B. Vitamin K C. protamine sulfate D. idarucizumab

B. Vitamin K Warfarin is simliar to vitamin K in structure and therefore acts as a competitive antagonist to hepatic use of vitamin K. Conversely vitamin K serves as the antidote for warfarin.

Which patient would be least likely to be prescribed clopidogrel? A) A 72-year-old patient with a history of CAD B) A 52-year-old patient with a recent angioplasty C) A 76-year-old patient with osteoporosis D) A 64-year-old patient with PVD

C) A 76-year-old patient with osteoporosis

What is the action of thrombolytics? A) Decrease blood coagulability- no effect on existing thrombus B) Prevent platelet aggregation at the site of blood vessel injury C) Destroy thrombi that are blocking blood vessels D) Prevent destruction of fibrin- promote clot formation

C) Destroy thrombi that are blocking blood vessels

A nurse is caring for a client who has AFib and a new prescription for dabigatran. Which of the following medications is prescribed concurrently to treat an adverse effect of dabigatran? A) Vitamin K B) Protamine C) Omeprazole D) Probenecid

C) Omeprazole

Which classification of prescriptions does the nurse anticipate for the client being treated for a thromboembolic​ disorder? ​Note: Credit will be given only if all correct choices and no incorrect choices are selected. Select all that apply. A. Hemostatics B. Thrombolytics C. Anticoagulants D. Antiplatelet agents E. Clotting factor concentrates

C. Anticoagulants D. Antiplatelet agents

The nurse notes the blood pressure and red blood cell​ (RBC) count of a client receiving heparin is low. Which should the nurse suspect has​ occurred? A. Clot formation B. Dehydration C. Internal bleeding D. Decreased activated partial thromboplastin time​ (aPTT)

C. Internal bleeding

Which additional prescribed treatment should the nurse anticipate for the client prescribed heparin​ therapy? A. Weekly weights B. Low vitamin K diet C. Obtaining an aPTT D. Advil as needed​ (PRN) for headaches

C. Obtaining an aPTT

Which prescription should the nurse anticipate for a client that has overdosed on Clopidogrel​ (Plavix)? A. Whole blood transfusion B. Vitamin K C. Platelet transfusion D. Protamine sulfate

C. Platelet transfusion

Which describes the purpose of​ fibrinolysis? A. Produce enzymes B. Stop blood flow C. Remove a blood clot D. Increase blood flow

C. Remove a blood clot

The nurse is reviewing the process of hemostasis after an injury with a client. Which should the nurse identify as the initial event in this​ process? A. Plasma proteins convert to active forms. B. Platelets become sticky. C. The vessel spasms. D. Von​ Willebrand's factor is activated.

C. The vessel spasms.

Which laboratory study is used to evaluate the proper dosage for heparin​ therapy? A. Serum heparin levels B. Sedimentation rate C. aPtt D. Complete blood count

C. aPtt

Which priority question should the nurse ask a client suspected of experiencing a stroke that is prescribed alteplase​ (Activase). A. ​"Are you currently being treated for​ hypertension?" B. ​"Do you have any other medical​ conditions?" C. ​"Do you know what time the stroke​ occurred?" D. ​"Do you take any other​ prescriptions?"

C. ​"Do you know what time the stroke​ occurred?"

The nurse has provided a client education about enoxaparin​ (Lovenox). Which statement made by a client indicates an understanding of the​ information? A. ​"Enoxaparin inhibits the synthesis of​ prostaglandins." B. ​"Enoxaparin increases the flexibility of my blood​ cells." C. ​"Enoxaparin increases the time it takes for me to form a​ clot." D. ​"Enoxaparin dissolves small clots so I​ won't have a​ stroke."

C. ​"Enoxaparin increases the time it takes for me to form a​ clot."

To compensate for clopidogrel's slow onset of action, the nurse should anticipate implementing which intervention? A. Temporarily supplementing the client's medication regimen with low-molecular-weight heparin B. Administering the first two to three doses of clopidogrel intravenously C. Administering a one-time loading dose that is higher than the maintenance dose D. Simultaneously administrating warfarin and clopidogrel

C. Administering a one-time loading dose that is higher than the maintenance dose Effective dose-dependent prevention of platelet aggregation can be seen within 2 hours of a single oral dose of clopidogrel, but the onset of action is slow, so a loading dose of 300 to 600 mg is usually administered. None of the other options effectively addresses the drug's slow onset of action.

The healthcare provider prescribes pentoxifylline (Trental) to a client, who asks the nurse how the medication works. Which response by the nurse is most accurate? A. By dissolving clots in the blood vessels B. By decreasing platelet aggregation C. By reducing the viscosity of red blood cells D. By increasing platelet production

C. By reducing the viscosity of red blood cells

Which information will the nurse include when teaching a patient about warfarin therapy? A. Increase the amount of green, leafy vegetables in your diet. B. Rinse your mouth instead of brushing your teeth. C. Follow up with laboratory tests such as PT or INR to regulate warfarin dose. D. Use a new razor blade each time you shave.

C. Follow up with laboratory tests such as PT or INR to regulate warfarin dose. (Laboratory tests such as PT or INR are performed to regulate warfarin (Coumadin) dose. The patient should avoid consuming large amounts of green, leafy vegetables; broccoli; legumes; soybean oil; coffee; tea; cola; excessive alcohol, and certain nutritional supplements such as coenzyme Q10. Patients are encouraged to perform oral hygiene and use a soft tooth brush to prevent gums from bleeding. Patients should be instructed to use an electric razor when shaving.)

Alteplase (Activase) is prescribed for a client with an acute episode of stroke. Which client evaluation is the nurse's priority action? A. Monitoring aPTT and platelet count B. Monitoring PT and INR serum levels C. Monitoring level of consciousness (LOC) D. Monitoring injection sites

C. Monitoring level of consciousness (LOC)

A direct thrombin inhibitor (DTI) has been added to a client's medication regimen to treat the onset of acute coronary syndrome. The nurse should anticipate administration by what route? Select all that apply. A. Subcutaneous B. Intramuscular C. Oral D. Intravenous E. Transdermal

C. Oral D. Intravenous Both parenteral and oral DTIs are available. The medication is not available in any of the other administration options.

A patient is prescribed aspirin, 81 mg, and clopidogrel. The nurse identifies the drug classification of clopidogrel as A. anticoagulant. B. thrombotic inhibitor. C. antiplatelet. D. thrombolytic.

C. antiplatelet.

The nurse is reviewing the laboratory results of a hospitalized patient receiving intravenous heparin therapy for pulmonary embolism. The activated partoial thromboplastin time (aPTT) is 38 seconds (control 28 seconds). The nurse should: A. not give the next dose because etc level is too high B. continue the present order because the level is appropriate C. notify the health care provider that the aPTT is low and anticipate orders to increase the dose D. request an order for warfarin now that the patient is heparinized

C. notify the health care provider that the aPTT is low and anticipate orders to increase the dose During heparin therapy, the aPTT should be maintained at approximately 1.502.5 times the control or baseline value. The normal control value is 25-35 seconds, and therapeutic values are 45-70 seconds approximately. A, C,

In developing a safe plan of care, the nurse recognizes that which of the following agents is the antidote for heparin? A. protamine zinc B. Vitamin K C. protamine sulfate D. Vitamin D

C. protamine sulfate Protamine sulfate is an antidote for standard heparin and low molecular weight heparin.

The nurse is teaching a client about the heparin which has been prescribed for a thrombus formation. Which information should the nurse prioritize in this teaching? Select all that apply. Clotting time returns to normal within 4 hours. Maximum effect occurs within 10 minutes. Onset of action is almost immediate. It causes fewer adverse reactions than the oral form. It is preferably given intramuscularly.

Clotting time returns to normal within 4 hours. Maximum effect occurs within 10 minutes. Onset of action is almost immediate. Parenteral heparin results in an almost immediate onset of action with a maximum effect within 10 minutes, but clotting returns to normal within 4 hours unless subsequent doses are given. It is preferably given subcutaneously or intravenously. The parenteral form has more potential adverse reactions than the oral form.

What is the action of antifibrinolytics? A) Decrease blood coagulability- no effect on existing thrombus B) Prevent platelet aggregation at the site of blood vessel injury C) Destroy thrombi that are blocking blood vessels D) Prevent destruction of fibrin- promote clot formation

D) Prevent destruction of fibrin- promote clot formation

Which route of administration should the nurse anticipate to use for a​ client's prescribed enoxaparin​ (Lovenox)? A. Administer the prescription via slow intravenous​ (IV) push. B. Administer the prescription intramuscularly into the thigh. C. Administer the prescription orally. D. Administer the prescription into the​ abdomen, subcutaneously.

D. Administer the prescription into the​ abdomen, subcutaneously.

Which adverse effect should the nurse instruct the client to monitor for while receiving warfarin​ sodium? A. Headache B. Pain C. Rash D. Bleeding

D. Bleeding

The nurse notes a​ client's warfarin​ (Coumadin) level is 7​ mcg/mL. Which action should the nurse​ take? A. Continue the treatment and monitor the client. B. Administer protamine sulfate and hold the next dose of warfarin​ (Coumadin). C. Hold the next dose of warfarin​ (Coumadin) and contact the healthcare provider. D. Hold the next dose of warfarin​ (Coumadin) and request an international normalized ratio​ (INR).

D. Hold the next dose of warfarin​ (Coumadin) and request an international normalized ratio​ (INR).

Which describes the primary action of anticoagulant​ therapy? A. Dissolve blood clots B. Decrease the prothrombin time C. Increase the number of platelets D. Prevent the formation of blood clots

D. Prevent the formation of blood clots

Which prescription should the nurse prepare to administer for a client experiencing a warfarin sodium​ overdose? A. Protamine sulfate B. Heparin C. Aspirin D. Vitamin K

D. Vitamin K

A client takes a nonsteroidal anti-inflammatory drug (NSAID) daily to manage the pain of arthritis. What response should the nurse make to the client's concern that another medication, like aspirin, should also be prescribed to minimize the risk for throm A. ''Adding an aspirin to your medication regimen is likely to help prevent clot formation.'' B. ''An NSAID's antiplatelet effect is minimal, but increasing the dosage would be helpful.'' C. ''I would suggest discussing this with your primary health care provider because NSAIDs have no antiplatelet effects.'' D. ''The antiplatelet effects of NSAIDs are effective in minimizing that risk.''

D. ''The antiplatelet effects of NSAIDs are effective in minimizing that risk.'' NSAIDs and many other aspirin-related drugs inhibit cyclooxygenase reversibly. Their antiplatelet effects subside when the drugs are eliminated from the circulation, and the drugs usually are not used for antiplatelet effects. However, clients who take an NSAID daily (e.g., for arthritis pain) may not need to take additional aspirin for antiplatelet effects. An increase in dosage would need to be discussed with a primary health care provider.

A client diagnosed with intermittent claudication has been prescribed cilostazol for its management. Medication education by the nurse should include informing the client that it may take a maximum of how long for symptom improvement to be achieved? A. 72 hours B. 4 weeks C. 5 months D. 12 weeks

D. 12 weeks Cilostazol is indicated for management of intermittent claudication. Symptoms usually improve within 2 to 4 weeks but may take as long as 12 weeks.

The nurse completes a physical assessment on a client receiving heparin therapy for deep vein thrombosis (DVT). The client complains of severe lumbar pain. What is the most appropriate action by the nurse? A. Reposition the client to promote comfort. B. Document the finding, and report it to the next nurse. C. Administer pain medication as prescribed. D. Evaluate further; this could indicate a complication of drug therapy.

D. Evaluate further; this could indicate a complication of drug therapy.

The client is prescribed warfarin. His INR is 5.2. At what level is this dose? Subtherapeutic Elevated Within prescribed limits Therapeutic

Elevated Explanation: Warfarin dosage is regulated according to the INR (derived from the prothrombin [PT] time), for which a therapeutic value is between 2.0 to 3.0 in most conditions. A therapeutic PT value is approximately 1.5 times the control, or 18 seconds.

A nurse is reviewing lab values of a client who is on warfarin (Coumadin) drug therapy. Which lab finding should the nurse report immediately to the healthcare provider? A. aPTT 48 seconds B. Bleeding time 7 seconds C. Platelet count 140,000 D. INR 1.2

D. INR 1.2

A nurse is caring for a client with epidural anesthesia following back surgery. The client is also receiving subcutaneous heparin 5,000 units twice daily for deep vein thrombosis (DVT) prophylaxis. What is the nurse's priority action? A. Monitor dose of the epidural anesthesia. B. Monitor effectiveness of pain management. C. Monitor the client for respiratory depression. D. Monitor the client's neurological status.

D. Monitor the client's neurological status.

By what means does warfarin achieve the desired anticoagulant effects? A. Dissolving microclots that develop in the atria and ventricles B. Preventing plasminogen from binding with fibrin C. Preventing platelets from aggregating on vessel walls D. Preventing synthesis of clotting factors in the liver

D. Preventing synthesis of clotting factors in the liver Warfarin acts in the liver to prevent synthesis of vitamin K-dependent clotting factors (i.e., factors II, VII, IX, and X). None of the other options accurately describe the action of warfarin.

A patient visits an outpatient clinic. The patient has been noncompliant with anticoagulation therapy and states, "I don't like having to have blood work all of the time." The nurse anticipates prescription of which medication? A. Abciximab B. Tirofiban C. Eptifibatide D. Rivaroxaban

D. Rivaroxaban (Two oral anticoagulants form a new anticoagulant category called Xa inhibitors. Rivaroxaban (Xarelto) was FDA-approved in July 2011, and apixaban (Eliquis) was FDA-approved in December 2012. These drugs do not require routine coagulation monitoring and are given q.d. or b.i.d. The other medications are administered intravenously.)

The nurse explains to a patient that aspirin suppresses blood clotting by A. inactivating thrombin B. promoting fibrin degradation C. decreasing synthesis of clotting factors D. decreasing platelet aggregation

D. decreasing platelet aggregation Aspirin, a cyclooxyrgenase inhibitor has potent anti platelet effects. The drug inhibits the production of thromboxoane, which under normal circumstances binds platelet molecules together to form a plug over damaged blood vessels walls.

The nurse would anticipate administering fondaparinux to a client with which condition? DVT treatment myocardial infarction DVT prophylaxis PE treatment

DVT prophylaxis Explanation: Fondaparinux is indicated for DVT prophylaxis. Enoxaparin is used to treat DVT and in PE treatment as well as unstable angina/non-Q-wave myocardial infarction (MI).

When the nurse administers warfarin, it is expected that the drug will have what effect on the body? Increase in prothrombin Decrease in production of vitamin K-dependent clotting factors Increase in vitamin K-dependent factors in the liver Increase in procoagulation factors

Decrease in production of vitamin K-dependent clotting factors Explanation: Warfarin, an oral anticoagulant drug, causes a decrease in the production of vitamin K-dependent clotting factors in the liver. The eventual effect is a depletion of these clotting factors and a prolongation of clotting times. It is used to maintain a state of anticoagulation in situations in which the client is susceptible to potentially dangerous clot formation. It does not increase prothrombin, vitamin K-dependent factors in the liver, or procoagulation factors.

A patient is prescribed warfarin to prevent the formation and extension of thrombus. The patient also takes a diuretic for the treatment of cardiac problems. What effect of interaction between the two drugs should the nurse monitor in the patient? Increased absorption of diuretics Decreased effectiveness of the anticoagulant Increased absorption of the anticoagulant Increased effectiveness of diuretics

Decreased effectiveness of the anticoagulant Explanation: The nurse should monitor for decreased effectiveness of warfarin as an effect of the interaction between the anticoagulant and the diuretic. The nurse need not monitor for the increased effectiveness of diuretics, the increased absorption of the anticoagulant, or the increased absorption of diuretics in the patient.

A client is prescribed warfarin. The client also takes a diuretic for the treatment of cardiac problems. The nurse will monitor the client for which potential interaction? Decreased effectiveness of the anticoagulant Increased effectiveness of the diuretic Increased absorption of the diuretic Increased absorption of the anticoagulant

Decreased effectiveness of the anticoagulant Explanation: The nurse should monitor for decreased effectiveness of warfarin as an effect of the interaction between the anticoagulant and the diuretic. The nurse need not monitor for the increased effectiveness of the diuretic, the increased absorption of the anticoagulant, or the increased absorption of the diuretic in the client.

A patient with thrombotic stroke is administered ticlopidine. What adverse reaction should the nurse monitor for in the patient? Dyspepsia Hematoma Dyspnea Bradycardia

Dyspepsia Explanation: The nurse should monitor for dyspepsia in the patient who has been administered ticlopidine. Hematoma is the adverse reaction to heparin. Dyspnea is an adverse reaction to protamine sulfate and treprostinil. Bradycardia is the adverse reaction to protamine sulfate.

A client is admitted to the hospital with deep vein thrombosis. An infusion of heparin is established. What action best protects the client's safety? Ensure that protamine sulfate is readily available Ensure that the client's call light is easily accessible. Have two nurses independently monitor the client's heparin infusion. Keep a preloaded syringe of vitamin K in the room.

Ensure that protamine sulfate is readily available Explanation: The antidote for heparin is protamine sulfate, and it is prudent to have this on hand if a client is receiving an infusion of heparin. Vitamin K reverses the effect of warfarin. The administration of heparin must be established and monitored carefully, but it is not usual practice for two nurses to maintain the infusion independent of one another and could lead to errors and omissions. Clients' call lights should always be available, but this is less significant than ensuring the availability of the antidote.

There are two pathways that can be activated by the coagulation process. One pathway begins when factor XII is activated. The other pathway begins when there is trauma to a blood vessel. What are these pathways? Factor and trauma pathways Inner and outer pathways Clotting and bleeding pathways Extrinsic and intrinsic pathways

Extrinsic and intrinsic pathways Explanation: The coagulation process results from the activation of what has traditionally been designated the intrinsic or the extrinsic pathways. The intrinsic pathway, which is a relatively slow process, begins in the circulation with the activation of factor XII. The extrinsic pathway, which is a much faster process, begins with trauma to the blood vessel or surrounding tissues and the release of tissue factor, an adhesive lipoprotein released from the subendothelial cells. The terminal steps in both pathways are the same: the activation of factor X and the conversion of prothrombin to thrombin.

A client taking warfarin (Coumadin) tells the nurse she is having bleeding in the gums and teeth. When reviewing client history, the client states she has decided to try an herbal remedy. Which of the following is most likely the cause of increased bleeding? Ginger Vitamin D Echinacea Lavender

Ginger Explanation: Warfarin should not be combined with any of the following substances, because they may have additive or synergistic activity and increase the risk for bleeding: celery, chamomile, clove, dong quai, feverfew, garlic, ginger, ginkgo biloba, ginseng, green tea, onion, passionflower, red clover, St. John's wort, and turmeric.

While preparing a client for discharge, the nurse is teaching the client about the proper use of warfarin. Which food should the nurse tell the client to avoid? Citrus fruits Green leafy vegetables Eggs Dairy products

Green leafy vegetables Explanation: Since vitamin K decreases the effect of warfarin, a client taking the drug should avoid consuming large amounts of foods that contain the vitamin. Green leafy vegetables, such as kale and some types of lettuce, are high in vitamin K.

The nurse is preparing to administer protamine emergently to a client per instructions from the health care provider. The nurse concludes this is necessary due to an adverse reaction to which drug? Warfarin Clopidogrel Alteplase Heparin

Heparin An overdosage of any anticoagulant may result in uncontrolled bleeding in the client. In most cases, discontinuation of the drug is usually sufficient to correct overdosage; however, if the bleeding is severe there are antidotes. Protamine is used to treat overdose of heparin and low-molecular-weight heparins (LMWHs). Vitamin K is used to treat the overdosage of warfarin. Alteplase and clopidogrel do not require antidotes.

The nurse is caring for a postpartum client admitted to the intensive care unit with a diagnosis of disseminated intravascular coagulation (DIC). What is the drug of choice to treat this problem? Urokinase Warfarin Heparin Aspirin

Heparin Explanation: The treatment of choice for DIC is heparin, an anticoagulant. It prevents the clotting phase from being completed, thus inhibiting the breakdown of fibrinogen. It may also help avoid hemorrhage by preventing the body from depleting its entire store of coagulation factors. None of the other medications listed in this question are indicated for treatment of DIC and may, in fact, make the condition worse.

A 76-year-old client is receiving IV heparin 5,000 units q8h. An activated thromboplastin time (aPTT) is drawn 1 hour before the 8:00 AM dose; the aPTT is at 3.5 times the control value. What would be the most appropriate action? Give the dose and chart the results. Hold the dose and call the result to the health care provider. Give the next two doses at the same time. Check the client's vital signs and place him or her on bleeding precautions.

Hold the dose and call the result to the health care provider. Explanation: Evaluate for therapeutic effects of heparin—whole blood clotting time (WBCT) 2.5 to 3 times control or activated partial thromboplastin time (APTT) 1.5 to 3 times the control value. It would be inappropriate to give 2 doses at once; give the dose and chart the results; or check the vital signs and place them on bleeding precautions.

By what route will the nurse administer the antihemophilic agent to the client with hemophilia following a car accident? Oral Topical IV Sublingual

IV Explanation: All antihemophilic agents are administered IV and are not available for administration by any other route.

A postsurgical client is being sent home on enoxaparin. The nurse should describe what benefit of this medication? Stimulating production of certain clotting factors Dissolving any clots that form Inhibiting the formation of clots Enhancing the flow of blood in peripheral vessels

Inhibiting the formation of clots Explanation: Low molecular weight heparins inhibit thrombus and clot formation by blocking factors Xa and IIa. Because of the size and nature of the molecules, these drugs do not greatly affect thrombin, clotting, or the PT; therefore, they cause fewer systemic adverse effects. Enoxaparin does not dissolve existing clots or directly enhance blood flow. It does not stimulate the production of clotting factors.

A 59-year-old client with vascular disorders is prescribed cilostazol. The nurse instructs the client that it is for the treatment of what condition? Pulmonary embolism Deep-vein thrombosis Intermittent claudication Venous stasis

Intermittent claudication Explanation: This drug is intended for the treatment of intermittent claudication, which is an arterial disorder. Pulmonary embolism, deep-vein thrombosis, and venous stasis are all venous disorders.

Assessment of a client receiving an anticoagulant reveals abdominal pain; coffee ground emesis; black, tarry stool; hematuria; joint pain; and spitting or coughing up blood. The nurse interprets these findings as which adverse reaction? Hepatic impairment Hypertensive emergency Cancer Internal bleeding

Internal bleeding Signs and symptoms of internal bleeding may include abdominal pain; coffee-ground emesis; black, tarry stools; hematuria; joint pain; and spitting or coughing up blood. None of these symptoms indicate all types of cancer, hypertensive emergency, or hepatic impairment.

A client is to receive abciximab. The nurse would expect to administer the drug by which route? Intravenous Oral Intramuscular Subcutaneous

Intravenous Explanation: Abciximab is administered intravenously.

The nurse is preparing to administer an anticoagulant to several clients. The nurse should question this order if which disorder(s) is noted in the client's medical record? Select all that apply. Leukemia Hypotension Atrial fibrillation GI ulcers Tuberculosis

Leukemia GI ulcers Tuberculosis Anticoagulants are contraindicated in clients with known sensitivity to the drug, active bleeding (except when caused by DIC), hemorrhagic disease, tuberculosis, leukemia, uncontrolled hypertension, GI ulcers, recent eye or CNS surgery, aneurysms, and severe renal and hepatic disease and during pregnancy and lactation. They are also contraindicated in clients with a hypersensitivity to pork products. Hypotension may be an indication of a hypersensitivity or adverse reaction to argatroban and protamine. Clients with atrial fibrillation are often prescribed anticoagulants to help decrease the risk of thrombus formation.

The black box warning associated with warfarin concerns its risk of causing what condition? DIC Severe coagulopathy Major or fatal bleeding Hypotension

Major or fatal bleeding Explanation: The FDA has issued a black box warning for warfarin due to its risk of causing major or fatal bleeding.

A nurse is caring for a client undergoing warfarin therapy for the treatment of venous thrombosis. The nurse suspects that the client is experiencing an overdose based on which finding? Melena Gastric upset Abdominal cramping Skin rash

Melena Explanation: Symptoms of warfarin overdosage include melena (blood in the stool); petechiae (pinpoint-sized red hemorrhagic spots on the skin); oozing from superficial injuries, such as cuts from shaving or bleeding from the gums after brushing the teeth; or excessive menstrual bleeding. Skin rash is an adverse reaction to clopidogrel. Abdominal cramping is an adverse reaction to anisindione. Gastric upset is an adverse reaction to phytonadione (vitamin K).

A 75-year-old client presents to the health care provider's office with bleeding gums and multiple bruises. When the nurse reviews the client's drug history, the nurse finds that the client is prescribed aspirin 81 mg/d. What drug may cause increased bleeding when used in conjunction with the aspirin? Antidysrhythmics Antihypertensives Antibiotics NSAIDs

NSAIDs Explanation: NSAIDs, which are commonly used by older adults, also have antiplatelet effects. Clients who take an NSAID daily may not need low-dose aspirin for antithrombotic effects.

What values are used to monitor the effectiveness of warfarin therapy? PT and INR RBC Platelet count aPTT

PT and INR Explanation: Warfarin dose is regulated according to the international normalized ratio (INR), which is based on prothrombin time (PT). PT and INR are assessed daily until a stable daily dose is reached. Thereafter, PT and INR are determined every few weeks for the duration of warfarin therapy.

A client with intermittent claudication is prescribed cilostazol by the primary health care provider. The nurse will administer this drug cautiously if the client's history reveals which disorder? Myocardial infarction Intermittent claudication Pancytopenia Pulmonary emboli

Pancytopenia Explanation: The nurse should administer cilostazol with caution to clients with pancytopenia. Anticoagulants are used for the prevention and treatment of pulmonary emboli, the adjuvant treatment of myocardial infarction, and the treatment of intermittent claudication.

A client is receiving heparin. Which would the nurse use to monitor the effects of the drug? Partial thromboplastin time International normalized ratio Prothrombin time Vitamin K level

Partial thromboplastin time Explanation: Heparin's effectiveness is monitored by the results of the partial thromboplastin time. The INR and PT are used to monitor warfarin. Vitamin K is the antidote for warfarin, and levels are not monitored to evaluate the effects of any anticoagulant.

The nurse is scheduled to administer the following medications at 0900: heparin subcutaneous, cefazolin IVPB, and atenolol PO. The morning labs are platelets 150,000 mcL, WBC 10,000 mcL, and APTT 100 seconds. Which is the nurse's priority in this situation? Place the client in supine position with legs elevated for BP 88/50 mm Hg. Instruct client to use an electric razor and soft toothbrush. Call primary care provider with APTT, BP 88/50 mm Hg, and recent void of red urine. Determine if there has been an improvement in the infection.

Place the client in supine position with legs elevated for BP 88/50 mm Hg. Explanation: All the listed medications can interact with heparin, and increase the risk for bleeding. The nurse should place the client in supine position with legs elevated since the client is hypotensive, BP 88/50 mm Hg. Placing a client in this position will help increase the blood pressure by increasing the return of blood to the heart. Next, the nurse should call the primary care provider (PCP) about the abnormal APTT, BP 88/50 mm Hg, and the recent void of red urine. The PCP will provide orders regarding whether to hold heparin or administer a reduced dose of the anticoagulant later, and what to do about the atenolol since the blood pressure is low, and may order follow-up labs, hemoglobin and hematocrit, and urinalysis for blood. The other actions are appropriate when caring for a client receiving the list of medications listed, but are not as important considering that the client may be bleeding. When a client receives anticoagulation, the nurse should instruct the client to use an electric razor, and a soft toothbrush to decrease the risk for bleeding. The client is taking an antibiotic, so the nurse should determine if there is an improvement in the infection. The client should have body temperature return to normal, and a decrease in the white blood count, and the symptoms of the infection improve and disappear.

The nurse is conducting the ongoing assessment of a client who is prescribed an anticoagulant. Which assessment(s) will the nurse prioritize? Select all that apply. Stool analysis Blood count Urinalysis Ultrasound Platelet count

Platelet count Blood count Stool analysis A complete blood count, platelet count, and stool analysis for occult blood may be ordered periodically throughout anticoagulant therapy. An urinalysis and ultrasound are not needed to monitor the effects of the anticoagulant.

The client is admitted to the hospital after a suicide attempt. The nurse discovers that the client overdosed on aspirin. What does the nurse expect the health care provider to do? Prescribe a transfusion of platelets. Prescribe the antidote. Order an IV infusion of Ringer's lactate at 100 mL/h. Prescribe a transfusion of whole blood.

Prescribe a transfusion of platelets. Explanation: No antidote exists for the effects of aspirin or the adenosine diphosphate receptor antagonists, because both produce irreversible platelet effects; platelet transfusion may be required.

A male client is receiving heparin by continuous intravenous infusion. The nurse will instruct the client and family members to report what should it occur? Presence of blood in urine or stools Sudden occurrence of sleepiness and drowsiness A skin rash Dizziness

Presence of blood in urine or stools Explanation: The nurse should instruct the client and family members to report the presence of blood in urine or stools and any bleeding from the gums, nose, vagina, or wounds. The anticoagulation properties of heparin can sometimes result in abnormal bleeding. Sleepiness, drowsiness, skin rash, and dizziness are not commonly identified adverse effects of the drug.

The nurse is preparing to administer warfarin to a female client, who also uses oral contraceptives. Which assessment finding(s) should the nurse prioritize? Select all that apply. Subtherapeutic INR Supratherapeutic INR Calf pain and warmth Blood in the stool Bruising

Subtherapeutic INR Calf pain and warmth Coadministration of warfarin and oral contraceptives can result in a decreased anticoagulant effect, leading to subtherapeutic INR and increased chance of clotting (signs and symptoms of DVT or PE). Bruising, melena, and supratherapeutic INR are adverse reactions to anticoagulants.

A nurse is conducting an assessment on a client who has recently undergone exploratory laparotomy. The nurse notes the continued order for enoxaparin. The nurse predicts this is to accomplish which goal? Encourage cessation of bleeding Prevent coagulation cascade Prevent formation of a blood clot Repair damage to a blood vessel

Prevent formation of a blood clot Explanation: This client has an increased risk for DVT and PE related to the major abdominal surgery. Enoxaparin is given subcutaneously prophylactically to help prevent the formation of a thrombus or blood clot. A thrombus refers to the formation of a blood clot, sometimes from damage, in a vessel that impedes blood flow. Cessation of bleeding refers to hemostasis. The coagulation cascade is the series of events that occur in the formation of a blood clot to stop bleeding. Hemostasis and the coagulation cascade are normal processes in the body.

Which describes the action of thrombolytic​ prescriptions? A. Convert plasmin to plasminogen. B. Prevent the liver from making fibrin. C. Digest and remove preexisting clots. D.

Prevent thrombus formation. C. Digest and remove preexisting clots.

The pharmacology instructor is discussing heparin with the students. How would the instructor explain the action of heparin? Promotes the inactivation of factor VIII Binds to factor Xa Promotes the inactivation of clotting factors Binds to factor X

Promotes the inactivation of clotting factors Explanation: Heparin blocks the formation of thrombin from prothrombin. Heparin does not bind to factors X and Xa. Heparin does not inactivate factor VIII.

A nurse is caring for a 64-year-old female client who is receiving IV heparin and reports bleeding from her gums. The nurse checks the client's laboratory test results and finds that she has a very high aPTT. The nurse anticipates that which drug may be ordered? Ticlopidine Coumadin Protamine sulfate Alteplase

Protamine sulfate Explanation: If a client who receives IV heparin is found to be highly anticoagulated, protamine sulfate may be prescribed. Protamine sulfate, which is a strong base, reacts with heparin, which is a strong acid, to form a stable salt, thereby neutralizing the anticoagulant effects of heparin. Protamine sulfate does not produce the same effects for coumadin, alteplase, or ticlopidine.

The nurse is assessing the medical record of a client who is prescribed warfarin. After the client reports occasionally using herbal supplements, which herb(s) should the nurse caution the client to avoid? Select all that apply. Ginger St. John's wort Arnica Aloe vera Ginkgo biloba

St. John's wort Ginkgo biloba Ginger Warfarin, a drug with a narrow therapeutic index, has the potential to interact with many herbal remedies. For example, warfarin should not be combined with any of the following substances, because they may have additive or synergistic activity and increase the risk for bleeding: celery, chamomile, clove, dong quai, feverfew, garlic, ginger, ginkgo biloba, ginseng, green tea, onion, passionflower, red clover, St. John's wort, and turmeric. Aloe vera is often used to treat burns. Arnica is often used to sprains and bruising.

A 59-year-old, who is receiving streptokinase for a myocardial infarction, passes a large tarry-looking stool. After the nurse notifies the health care provider, what will the nurse expect to do? Reduce the streptokinase and administer vitamin K Stop the streptokinase and administer protamine sulfate Stop the streptokinase and administer aminocaproic acid (Amicar) Reduce the streptokinase and administer heparin

Stop the streptokinase and administer aminocaproic acid (Amicar) Explanation: The hemostatic drug that is used systemically is aminocaproic acid . Aminocaproic acid is used to stop bleeding caused by overdoses of thrombolytic agents. The other options are incorrect.

A client is to receive enoxaparin. The nurse would administer this drug by which route? Intravenous infusion Orally Intramuscular injection Subcutaneous injection

Subcutaneous injection Explanation: Enoxaparin is administered via subcutaneous injection. Intramuscular (IM) administration is avoided because of the possibility of the development of local irritation, pain, or hematoma (a collection of blood in the tissue). Intravenous infusions are used with several anticoagulants such as heparin. Warfarin is given orally.

A stable daily dose of warfarin is reached when which parameter is achieved? The PTT is within the therapeutic range. The INR is between 4 and 5 The INR is between 1 and 2, and the dose does not cause bleeding. The PT and INR are within their therapeutic ranges, and the dose does not cause bleeding.

The PT and INR are within their therapeutic ranges, and the dose does not cause bleeding. Explanation: When warfarin is started, PT and INR should be assessed daily until a stable daily dose is reached (the dose that maintains PT and INR within therapeutic ranges and does not cause bleeding).

The nurse administers clopidogrel appropriately to the client for what purpose? Maintaining the patency of grafts Dissolving a pulmonary embolus and improving oxygenation Treating peripheral artery disease Preventing emboli from valve replacements

Treating peripheral artery disease Explanation: Clopidogrel is used to inhibit platelet aggregation, decreasing the formation of clots in narrowed or injured blood vessels like those found in peripheral artery disease. Maintaining the patency of grafts or preventing emboli from valve replacements would be accomplished using an anticoagulant. Dissolving emboli would be accomplished using streptokinase or a similar enzyme to stimulate the conversion of plasminogen to plasmin.

The nurse is teaching a young female about the prescribed anticoagulant. The nurse determines the session is successful when the client correctly indicates they will apply which recommendation? Take the drug on an empty stomach. Take the drug with a glass of milk. Avoid caffeinated drinks. Use a reliable contraceptive.

Use a reliable contraceptive. Explanation: The nurse should instruct the female client to use a reliable contraceptive to prevent pregnancy. The nurse need not instruct the client to avoid caffeinated drinks, take the drug on an empty stomach, or take the drug with a glass of milk.

When describing the clotting process, which step would the nurse identify as the first reaction to occur with injury to a blood vessel? Vasoconstriction Thrombin formation Platelet aggregation Release of factor XI

Vasoconstriction Explanation: The first reaction to a blood vessel injury is local vasoconstriction. In addition, injury then exposes blood to the collagen and other substances under the endothelial lining of the vessel, causing platelet aggregation. Release of factor XI occurs in response to activation of the Hageman factor. Thrombin formation occurs at the end of the intrinsic pathway.

The physician reorders dalteparin for the client. The pharmacy states that the drug is not available. The nurse calls the physician and asks if dalteparin can be replaced with another LMWH. What does the nurse expect to be the physician's response? a) "The drugs are not interchangeable; have the pharmacy obtain the drug ordered." b) "The drugs are interchangeable; the pharmacy can substitute another LMWH." c) "I'll change the order to enoxaparin." d) "I'll change the drug to Lovenox."

a) "The drugs are not interchangeable; have the pharmacy obtain the drug ordered." Rationale: Currently available LMWHs (dalteparin, enoxaparin) differ from standard heparin and each other and are not interchangeable.

After teaching a group of students about antiplatelet agents, the instructor determines that the teaching was successful when the students identify which drug as blocking the production of platelets in the bone marrow? a) Anagrelide b) Cilostazol c) Clopidogrel d) Ticlopidine

a) Anagrelide Rationale: Anagrelide blocks the production of platelets in the bone marrow. Cilostazol inhibits platelet aggregation and adhesion by blocking receptor sites on the platelet membrane preventing platelet-platelet interaction or the interaction of platelets with other clotting chemicals. Clopidogrel inhibits platelet aggregation and adhesion by blocking receptor sites on the platelet membrane preventing platelet-platelet interaction or the interaction of platelets with other clotting chemicals. Ticlopidine inhibits platelet aggregation and adhesion by blocking receptor sites on the platelet membrane preventing platelet-platelet interaction or the interaction of platelets with other clotting chemicals.

Which of the following is classified as an antiplatelet agent? (Choose one) a) Clopidogrel (Plavix) b) Warfarin (Coumadin) c) Enoxaparin (Lovenox) d) Alteplase (Activase) e) Phytonadione (Mephyton)

a) Clopidogrel (Plavix) Rationale: Plavix is an antiplatelet agent. Activase is a thrombolytic agent. Mephyton is an anticoagulant antagonist. Coumadin is an oral anticoagulant. Lovenox is a parenteral anticoagulant.

An 18-year-old woman who takes birth control pills and is a heavy smoker is scheduled for abdominal surgery. Which of the following post-surgical nursing interventions would be most important to prevent thrombus formation? a) Encourage active range-of-motion exercises every 4 hours. b) Maintain the client on strict bed rest. c) Ensure that the the client smokes only in a designated area. d) Give the client Motrin to control minor pain. Answer: Encourage active range-of-motion exercises every 4 hours.

a) Encourage active range-of-motion exercises every 4 hours.

Ms. Carter is discharged from the hospital after a myocardial infarction. The physician prescribes continuance of the drug clopidogrel at home to help prevent future myocardial infarction (MI). Five days after discharge, she contacts the physician's office with complaints of low-grade fever and painful GI distress. What important adverse effect(s) should the nurse instruct the patient to monitor for? a) GI ulcers and bleeding b) MIs, called heart attacks c) Tissue necrosis d) CVAs, called strokes

a) GI ulcers and bleeding

You are caring for a patient who is going home on warfarin (Coumadin). You will schedule the patient for what following laboratory tests to help evaluate therapeutic effects of the drug? a) Prothrombin time (PT) and international normalized ratio (INR) b) Prothrombin time (PT) only c) Prothrombin time (PT) and activated partial thromboplastin time (APTT) d) International normalized ratio (INR) only

a) Prothrombin time (PT) and international normalized ratio (INR)

Ms. Main is sent home with subcutaneous heparin after a total hip replacement. What symptom would indicate a serious drug reaction? a) Tarry stools b) Headache c) Stomach pain d) Hypotension

a) Tarry stools Tarry stools would be an indication of gastrointestinal bleeds. The most common adverse effect of heparin is bleeding.

Which of the following should the nurse complete during the pre-administration assessment for a client with a DVT? Select all that apply: a) Test for a positive Homans' sign. b) Assess gait and balance. c) Assess pain. d) Examine extremity for skin temperature. e) Check for pedal pulse.

a) Test for a positive Homans' sign. c) Assess pain. d) Examine extremity for skin temperature. e) Check for pedal pulse. Rationale: Pre-administration assessment for a client with a DVT should include checking for a pedal pulse, examining the extremity for color and skin temperature, assessing for pain, and checking for a positive Homan' sign.

The client is diagnosed with hepatitis A, diabetes type 1, and portal hypertension. He develops a DVT, and the physician prescribes warfarin. The nurse is concerned, for what reason? a) The client is more likely to experience bleeding. b) The client is at risk for further liver impairment. c) The client is at risk for hyperglycemic episodes. d) The client is less likely to achieve a therapeutic dose.

a) The client is more likely to experience bleeding. Rationale:Warfarin is more likely to cause bleeding in clients with liver disease because of decreased synthesis of vitamin K. In addition, warfarin is eliminated only by hepatic metabolism and may accumulate with liver impairment.

An instructor is describing the drug drotrecogin alfa to a group of students. The instructor determines that the students need additional teaching when they state which of the following? a) The drug is associated with a high risk for embolism. b) The drug is administered by intravenous infusion for a total of 4 days. c) The drug is fairly expensive, possibly limiting its used. d) The drug is used primarily for adults with severe sepsis.

a) The drug is associated with a high risk for embolism. Rationale: Drotrecogin alfa is approved for use in adults with severe sepsis who are at high risk for death. The usual dose is given by intravenous infusion for a total of 96 hours. It is associated with a high risk for bleeding, which requires extremely close client monitoring. It is a very expensive drug, which may be a factor in limiting its use.

The squad just delivered to your ED a 73-year-old man with crushing chest pain. He has a history of cardiovascular disease. Onset of symptoms is within the last 60 minutes. What medication would you expect the physician to order for his thromboembolic disorder? a) Thrombolytic drugs b) Antiplatelet drugs c) Direct thrombin inhibitor drugs d) Anticoagulant drugs

a) Thrombolytic drugs Rationale: The main use of thrombolytic agents is for management of acute, severe thromboembolic disease, such as myocardial infarction or pulmonary embolism.

What laboratory value will the nurse prioritize when providing care for a client prescribed intravenous heparin? aPTT D-dimer platelet count factor XIII levels

aPTT Explanation: Prescribers use the activated partial thromboplastin time (aPTT), which is sensitive to changes in blood clotting factors, except factor VII, to regulate heparin dosage. D-dimer test is used to help rule out deep vein thrombosis (DVT) and pulmonary embolism (PE). A platelet count measures how many platelets are present in the blood. Platelets are parts of the blood that help the blood clot. Factor XIII levels are assessed when diagnosing/managing hemophilia A.

A nurse has an order to administer heparin. Before initiating this therapy, a priority nursing assessment will be the client's: blood glucose level. heart rate and pulse. sodium and potassium levels. aPTT.

aPTT. Explanation: Before initiating therapy, it is important to review the client's aPTT, hematocrit, and platelet count. These tests provide baseline information on the client's blood clotting abilities and identify conditions that may cause heparin therapy to be contraindicated. The client's heart rate, pulse, electrolyte, and blood sugar levels would not be priority nursing assessments.

A client who received a tissue plasminogen activator (tPA) begins to bleed. Which medication will the nurse provide as prescribed to control the bleeding? aminocaproic acid protamine sulfate vitamin K idarucizumab

aminocaproic acid Explanation: Aminocaproic acid is used to control bleeding caused by overdoses of thrombolytic agents and is an antidote for tissue plasminogen activator (tPA). Vitamin K is the antidote for an overdose of warfarin. Idarucizumab is the reversal agent for dabigatran etexilate. Protamine sulfate is the treatment for a heparin overdose.

A client with acute coronary syndrome is prescribed clopidogrel. Which additional medication will the nurse expect to be prescribed for this client? aspirin acetaminophen ibuprofen digoxin

aspirin Explanation: Clopidogrel is an adenosine diphosphate receptor antagonist used to prevent platelet aggregation. In acute coronary syndrome, clopidogrel is given with aspirin to prevent thrombus formation. Clopidogrel does not need to be given with digoxin, ibuprofen, or acetaminophen.

You know that your teaching about warfarin (Coumadin) is successful if your client makes what statement? a) "If I miss a dose, I will take 2 pills the next day." b) "I will ask my doctor about herbal supplements." c) "I will increase the dark-green leafy vegetables in my diet." d) "I will take a multivitamin daily."

b) "I will ask my doctor about herbal supplements." Rationale: Many commonly used herbs and supplements have a profound effect on drugs used for anticoagulation. Options A, C, and D would tell you the patient needs more teaching.

The nurse is reviewing the coagulation studies of a client who is receiving a heparin infusion. The client's baseline partial thromboplastin time is 32 seconds. Which result would indicate therapeutic effectiveness? a) 32 seconds b) 64 seconds c) 40 seconds d) 96 seconds

b) 64 seconds Rationale: Therapeutic range for heparin would be 1.5 to 2.5 times the client's baseline. For a baseline value of 32 seconds, this would range from 48 to 80 seconds. A value of 64 seconds would be considered therapeutic.

Which of the following statements are true in regards to anticoagulants? Select all that apply: a) Anticoagulants dissolve existing thrombi. b) Anticoagulants prevent extension of a thrombus. c) Anticoagulants can reverse the damage caused by a thrombus. d) Anticoagulants thin the blood. e) Anticoagulants prevent formation of a thrombus.

b) Anticoagulants prevent extension of a thrombus. e) Anticoagulants prevent formation of a thrombus. Rationale: Anticoagulants can prevent the formation and extension of a thrombus, but have no direct effect on an existing thrombus or reverse any of the damage from that thrombus. Although clients often refer to anticoagulants as blood thinners, they do not actually thin the blood.

The client presents to the physician's office because the physician is unable to regulate his Coumadin dosage. During the interview, the nurse finds that the client began taking what substance, which might increase the effects of the warfarin? a) Red meat to increase his protein intake b) Ginseng to improve his energy c) Milk to increase his calcium level and prevent cramps d) Chamomile tea to help him sleep

b) Ginseng to improve his energy Rationale: Herbs commonly used that may increase the effects of warfarin include alfalfa, celery, clove, feverfew, garlic, ginger, ginkgo biloba, ginseng, and licorice.

Ms. Adams is started on warfarin therapy. The patient asks you why she is not on heparin therapy any longer. What differentiates heparin from warfarin? a) Heparin is an anticoagulant, while warfarin is a thrombolytic. b) Heparin is administered parenterally, while warfarin is administered orally. c) Heparin causes more serious adverse effects than warfarin. d) Heparin is used to treat thrombosis, while warfarin treats heart attacks.

b) Heparin is administered parenterally, while warfarin is administered orally. Heparin is administered parenterally, while warfarin is administered orally. Warfarin is administered after heparin therapy to complete treating a thrombus or embolism. Clients receiving heparin therapy will begin taking warfarin before they discontinue heparin. This overlap allows the warfarin to reach a therapeutic level before heparin is discontinued. However, both drugs are used to treat thrombosis and thromboembolic disorders and have equally serious adverse effects, such as bleeding. Anticoagulants, such as warfarin and heparin, prevent new clot formation or extension of an existing clot, while thrombolytics dissolve existing blood clots.

You are sending a patient home on heparin therapy. What is important when you teach a patient to administer heparin subcutaneously? a) Leave a small air bubble in the syringe to lock in the dose b) Insert the needle at a 45- to 90-degree angle c) Gently massage the site after the injection d) Use the Z-track method

b) Insert the needle at a 45- to 90-degree angle

A client is receiving warfarin. The nurse would expect to administer this drug by which route? a) Intramuscular b) Oral c) Intravenous d) Subcutaneous

b) Oral Warfarin is administered orally.

The physiology instructor is discussing blood clots. What would the instructor tell the students is the basis for the clot-dissolving system? a) Thrombin b) Plasminogen c) Fibrinolysin d) Hageman factor

b) Plasminogen Rationale: Plasminogen is the basis for the clot-dissolving system. It is converted to plasmin (Fibrinolysin) by several factors including Hageman Factor, which is factor XII that is found in circulating blood. Activated thrombin breaks down fibrinogen to form fibrin threads, which form a clot inside the blood vessel.

The physician discovers a clot in the client's left lower leg. The physician prescribes anticoagulant drugs to prevent formation of new clots and to achieve which other effect? a) Regulate PTT b) Prevent extension of clots already present c) Increase coagulation d) Regulate PT, INR

b) Prevent extension of clots already present Anticoagulant drugs are given to prevent formation of new clots and extension of clots already present.

A patient is admitted to the hospital with deep vein thrombosis. Heparin 10,000 units is administered Sub-Q. Which of the following drugs should always be available in the pharmacy if the patient begins to bleed? a) Vitamin K b) Protamine sulfate c) Antithrombin (Thrombate III) d) Desirudin (Iprivask)

b) Protamine sulfate

The nursing instructor is talking about anticoagulant therapy. The instructor explains that when administering warfarin (Coumadin) to a client with deep vein thrombophlebitis, the nurse monitors the PTT, PT, INR, and hematocrit. Which laboratory value would the instructor say indicates warfarin is at therapeutic levels? a) Partial thromboplastin time (PTT) 1.2 to 2.5 times the control b) Prothrombin time (PT) 1.2 to 1.5 times the control c) International normalized ratio (INR) of 3 to 4 d) Hematocrit of 32%

b) Prothrombin time (PT) 1.2 to 1.5 times the control Rationale: Evaluate for therapeutic effects of warfarin—prothrombin time (PT) 1.2 to 1.5 times the control value. Heparin, not warfarin, prolongs PTT. The INR may also be used to determine if warfarin is at a therapeutic level. An INR of 2 to 3 is considered therapeutic. Hematocrit does not provide information on the effectiveness of warfarin; however, a falling hematocrit in a client taking warfarin may be a sign of hemorrhage.

Administration of streptokinase (Streptase) will result in which of the following? Select all that apply: a) Prevent the formation of a thrombus b) Reopening of occluded blood vessels c) Decrease risk of internal bleeding d) Breakdown of existing thrombi e) Prevention of tissue necrosis

b) Reopening of occluded blood vessels d) Breakdown of existing thrombi e) Prevention of tissue necrosis Rationale: Streptokinase (Streptase) is a thrombolytic drug. Thrombolytic drugs break down existing thrombi, reopen blood vessels after occlusion, and prevent tissue necrosis.

Atherosclerosis can affect any organ or tissue but often involves specific locations in the body. Plaque sites contribute to the formation of a thrombus, which can partially or completely occlude the blood vessel. Angina can be caused by a thrombus forming in: a) The legs. b) The heart. c) The brain. d) The lungs.

b) The heart. In coronary arteries, a thrombus may precipitate myocardial ischemia (angina or infarction). Assessment would focus on the coronary system, rather than any of the other presented options.

Clopidogrel (Plavix) is used frequently for what? a) Preventing emboli from valve replacements b) Treatment of peripheral artery disease c) Dissolving a pulmonary embolus and improving oxygenation d) Maintaining the patency of grafts

b) Treatment of peripheral artery disease Rationale:Clopidogrel is used to inhibit platelet aggregation, decreasing the formation of clots in narrowed or injured blood vessels like those found in peripheral artery disease. Maintaining the patency of grafts or preventing emboli from valve replacements would be accomplished using an anticoagulant. Dissolving emboli would be accomplished using streptokinase or a similar enzyme to stimulate the conversion of plasminogen to plasmin.

A 70-year-old resident of a long-term care facility has experienced increasing pain, swelling, and redness to her lower leg over the past 12 hours, prompting the care providers at the facility to have the woman brought to the local emergency department. Diagnostic testing has confirmed deep vein thrombosis (DVT) and an IV infusion of alteplase (Activase) has been initiated. Unlike anticoagulants, a thrombolytic such as alteplase can: break down existing clots. prevent the formation of new clots. inhibit platelet aggregation. potentiate the action of vitamin K.

break down existing clots. Explanation: Unlike anticoagulants, thrombolytics are able to break down existing clots, not solely prevent the formation of new clots. Potentiation of vitamin K would exacerbate clotting and the pharmacodynamics of thrombolytics do not primarily affect platelet action.

A nurse is preparing to discharge a patient newly prescribed warfarin (Coumadin). While assessing the patient's knowledge of their drug, what would indicate that the patient needs further instruction concerning her drug therapy? a) "I drink a glass of wine about once a week." b) "I love to eat homegrown tomatoes in the summer." c) "I take aspirin for my arthritis." d) "I walk two miles a day."

c) "I take aspirin for my arthritis." Increased bleeding can occur if a salicylate is taken in combination with warfarin. The nurse will instruct the client to stop taking aspirin. Walking, taking vitamin C, and drinking an occasional glass of wine should not interfere with the therapeutic effects of warfarin.

Anticoagulant drugs prevent thromboembolic disorders. How does warfarin, one of the anticoagulant drugs, act on the body? a) Increases prothrombin b) Increases vitamin K-dependent factors in the liver c) Alters vitamin K reducing its ability to participate in the coagulation of the blood d) Increases procoagulation factors

c) Alters vitamin K reducing its ability to participate in the coagulation of the blood Rationale: Warfarin causes a decrease in the production of vitamin K-dependent clotting factors in the liver. The eventual effect is a depletion of these clotting factors and a prolongation of clotting times. It does not increase prothrombin, vitamin K-dependent factors in the liver, or procoagulation factors.

Bleeding resulting from the administration of heparin is treated with which of the following substances? a) Vitamin E b) Vitamin K c) Protamine sulfate d) Calcium gluconate

c) Protamine sulfate

The physician orders heparin for a 35-year-old female client. The nurse administers the drug only after confirming that the client a) is not hypertensive. b) does not have a urinary tract infection. c) does not have peptic ulcer disease. d) does not have asthma.

c) does not have peptic ulcer disease.

Ms. Kim is admitted to the hospital for treatment of a pulmonary embolism. Forty-eight hours after therapy has started, Ms. Kim reports bleeding from her gums when she brushes her teeth. The nurse notes this to be an adverse effect of the medication and contacts the physician. What orders can the nurse anticipate receiving from the physician to reverse the effects of the heparin? a) Administer protamine sulfate, at a dosage of 2 mg/kg. b) Administer procainamide sulfate, at a dosage of 2 mg/kg, for a maximum dose of 200 mg. c) Administer procainamide sulfate, at a dosage of 1 mg per 1,000 units of heparin. d) Administer protamine sulfate, at a dosage of 1 mg per 100 units of heparin, for a maximum dose of 100 mg.

d) Administer protamine sulfate, at a dosage of 1 mg per 100 units of heparin, for a maximum dose of 100 mg. Rationale: The protamine sulfate dose is based on the heparin dose: 1 mg of protamine sulfate per 100 units of heparin. No more than 100 mg of protamine sulfate should be given within a 2-hour period, because this drug can cause anticoagulation in its own right.

A 60-year-old patient experienced a sudden onset of chest pain and shortness of breath and was subsequently diagnosed with a pulmonary embolism in the emergency department. The patient has been started on an intravenous heparin infusion. How does this drug achieve therapeutic effect? a) By inhibiting platelet aggregation on vessel walls and promoting fibrinolysis b) By inhibiting the action of vitamin K at its sites of action c) By promoting the rapid excretion of vitamin K by the gastrointestinal mucosa d) By inactivating clotting factors and thus stopping the coagulation cascade

d) By inactivating clotting factors and thus stopping the coagulation cascade Heparin, along with antithrombin, rapidly promotes the inactivation of factor X, which, in turn, prevents the conversion of prothrombin to thrombin. Heparin does not achieve its therapeutic effect through the excretion or inhibition of vitamin K or by inhibiting platelet aggregation.

A 59-year-old patient is on warfarin therapy. On follow-up visits to the clinic, the nurse will assess the patient for which of the following? a) Presence of skin-related disorders b) Blood glucose level c) Presence of breathing disorders d) Intake of vitamin K

d) Intake of vitamin K

The primary health care provider orders a 300-mg dose of clopidogrel for a client. The client then is to receive 75 mg/day. The nurse interprets the 300-mg dose to be which type of dose? a) Treatment of stroke b) Maintenance dose c) Treatment of myocardial infarction d) Loading dose

d) Loading dose Clopidogrel 300 mg should only be administered as a single loading dose. Maintenance dose is medication taken on schedule to maintain a level or effect. Thrombolytic drugs are used to treat myocardial infarction and stroke caused by clot formation.

A client is to receive clopidogrel. The nurse would expect to administer this agent by which route? a) Subcutaneous b) Intravenous c) Intramuscular d) Oral

d) Oral

Which of the following would the nurse identify as the end of the intrinsic pathway? a) Vasoconstriction b) Platelet aggregation c) Release of factor XI d) Thrombin formation

d) Thrombin formation Rationale: Thrombin formation occurs at the end of the intrinsic pathway. The first reaction to a blood vessel injury is local vasoconstriction. In addition, injury then exposes blood to the collagen and other substances under the endothelial lining of the vessel causing platelet aggregation. Release of factor XI occurs in response to activation of the Hageman factor.

You know that your 48-year-old patient needs additional teaching regarding anticoagulants if he states that he will what? a) Report to the lab once a month b) Carry a Medic Alert card with him c) Use an electric razor d) Use aspirin for arthritis pain

d) Use aspirin for arthritis pain

The nurse is concerned that the physician does not order routine aPTTs when the client is receiving LMWH for thromboembolism prophylaxis. When the nurse calls the physician with the nurse's concern, what will be the physician's response? a) aPTTs should be drawn weekly. b) INRs should be drawn weekly. c) aPTTs should be drawn daily. d) aPTTs are not needed.

d) aPTTs are not needed.

A postsurgical client possesses numerous risk factors for venous thromboembolism, including a previous deep vein thrombosis. What drug would the nurse anticipate administering while this client recovers in the hospital? heparin clopidogrel cilostazol vitamin K

heparin Explanation: Heparin is frequently used to prevent postsurgical venous thromboembolism. Antiplatelet drugs do not have this indication, and vitamin K would increase the client's risks.

A client accidentally ingested extra doses of warfarin at home and now presents with an elevated INR and bleeding. Which medication will the nurse prepare as prescribed to administer along with vitamin K? idarucizumab aminocaproic acid prothrombin complex concentrate tranexamic acid

prothrombin complex concentrate Explanation: Urgent reversal of warfarin overdosage in adults with acute major bleeding can be accomplished with prothrombin complex concentrate (PCC). The drug should be administered as prescribed and concurrently with vitamin K to maintain factor levels once the effects of PCC have diminished. Idarucizumab is the antidote for dabigatran. Tranexamic acid and aminocaproic acid are used to stop bleeding caused by overdoses of thrombolytic agents.

The pharmacology instructor is discussing clopidogrel (Plavix). According to the instructor, this drug is frequently used for: prophylaxis of deep venous thrombosis in clients having hip replacement surgery. thrombosis prophylaxis in post-surgical clients. reduction of elevated platelet count. reduction of myocardial infarction in clients with atherosclerosis.

reduction of myocardial infarction in clients with atherosclerosis. Explanation: Clopidogrel (Plavix) is indicated for reduction of myocardial infarction, stroke, and vascular death in clients with atherosclerosis. These drugs work by decreasing the platelets' ability to stick together (aggregate) in the blood, thus forming a clot.

To monitor possible intermittent claudication, the nurse will assess for a thrombus lodging in what specific location? the brain the lungs the legs the heart

the legs In peripheral arteries, a thrombus in peripheral circulation may cause intermittent claudication (pain in the legs with exercise) or acute occlusion. Assessment would focus on the legs, rather than any of the other presented options.


Conjuntos de estudio relacionados

Intro to New Testament Ch. 6,7,8,11

View Set

Chapter 5 Unit 3: Listing Agreements

View Set

ARH 151 Test 3 Vocab 2.2, 4.10, 4.11

View Set

PN Adult Medical Surgical Online Practice 2020 A

View Set

Intro to business SmartBook Ch. 12, 11 ,and 7

View Set

Outdoor Pursuits Chapter Reviews

View Set